You are on page 1of 111

.

CHAPTER-04

MOTION IN ONE DIMENSION


4.1 FRAME OF REFERENCE : A frame of reference is the frame in which observer sits and makes
observation. To make observation we attach a Cartesian coordinates system. Cartesian coordinates
system consists of three mutually perpendicular axis, x, y and z meeting at common point O,
called origin
y

x
o

4.2 REST AND MOTION : If a body changes its position as time passes (with respect to reference
point) called in motion otherwise in rest.
Example : A man sitting inside moving train is at rest with respect to other passenger. But his
position change with respect to an observer on ground, So it is in motion with respect to an observer
on ground.
In fact rest and motion depends on the choice of frame (observer)
Different types of Motion :
(1) Depends on the coordinates system :
One dimensional Two dimensional Three dimensional
Motion of a body in a straight line is Motion of body in a plane is called Motion of body in a space is called
called one dimensional motion. two dimensional motion. three dimensional motion.
When only one coordinate of the When two coordinates of the position When all three coordinates of the
position of a body changes with time of a body changes with time then it is position of a body changes with time
then it is said to be moving one said to be moving two dimensionally. then it is said to be moving three
dimensionally. dimensionally.
e.g. . Motion of car on a straight road. e.g . Motion of car on a circular turn. e.g. . Motion of flying kite.

Motion of freely falling body. Motion of billiards ball and Motion of flying insect.
projectile motion
(2) Circular Motion and Rotatory Motion :
(i) Motion of a point mass on a circular path called circular motion.
(ii)Motion of a body about an axis passing through body is called rotatory motion. In rotatory
motion all the particles of a rigid body describes the same angular displacement about an axis
of rotation.
(3) Oscillatory Motion : When object repeats its motion about fixed point called oscillatory motion.
Kinematics [ 3 ]
4.3 POSITION VECTOR : In figure, a point A in space whose co-ordinates are (x, y, z), then position
vector is defined as
   
r  x i  y j  zk.
Its magnitude is given by
 y
r  x 2  y 2  z2
In plane (x, y) position vector is given by A

   r
r  x i y i
 x
and magnitude of r is
 z
r  x2  y2
4.4 DISPLACEMENT AND DISTANCE :
(1) Distance : It is the actual path length covered by a moving particle in a given interval of time.
(i) If a particle starts from A and reach to C through point B as shown in the figure.

4m

A 3m B

Then distance travelled by particle = AB + BC = 7m


(ii) Distance is a scalar quantity.
(iii) Dimension : [M0L1T0]
(iv) Unit : metre (S.I.)
(2) Displacement : Displacement is the change in position vector i.e., A vector joining initial to final
position.
(i) Displacement is a vector quantity
(ii) Dimension : [M0L1T0]
(iii) Unit : metre (S.I.)
(iv) In the above figure the displacement of the particle
  
AC  AB BC

 AC  (AB)2  (BC)2  2(AB)(BC)cos 90  5m
   
(v) If S1,S2 ,S3 ....Sn are the displacements of a body then the total (net) displacement is the
    
vector sum of the individuals. S  S1  S2  S 3 Sn

Corporate Office : B-58, Goal Building, Budha Colony, Patna-1, Help line : 9334594165/66/67 Website : www.goalinstitute.org
Kinematics [4]

IMPORTANT POINTS – 1
(a) The magnitude of displacement is equal to minimum possible distance between two positions.
So distance  |Displacement|.
(b) For a moving particle distance can never be negative or zero while displacement can be.
(Zero displacement means that body after motion has came back to initial position)
i.e., Distance > 0 but Displacement > = or < 0
Y
B
S

P
 
rB rAB
A

rA
X
(c) For motion between two points displacement is single valued while distance depends on
actual path and so can have many values.
(d) For a moving particle distance can never decrease with time while displacement can.
Decrease in displacement with time means body is moving towards the initial position.
(e) In general magnitude of displacement is not equal to distance. However, it can be so if the
motion is along a straight line without change in direction.
 
(f) If rA and rB are the position vectors of particle initially and finally.
  
Then displacement of the particle rAB  rB  rA and S is the distance travelled if the particle
has gone through the path APB.

Solved examples – 1
Ex.1. A mosquito flies from point A (–1, 2, 3)m to a point B (2, –1, –2)m, find the displacement of the
mosquito.
(1) 2 22 (2) 43 (3) 53 (4) 64
         
Soln. (2)  r  r2 – r1  (2 – (–1)) i  (–1 – 2) j  (–2 – 3)k  3 i – 3 j – 5k  |  r |  43
Ex.2. A particles starts from the origin, goes along the X-axis to the point (20m, 0) and then returns along
the same line to the point (–20m, 0). Then distance and displacement of the particle during the trip
(in m).
(1) 60 20 (2) 10, 20 (3) 30, 40 (4) 20, 10
Soln. (1) 60, 20
Ex.3. A man goes 10m towards North, then 20m towards East then displacement is
(1) 22.5m (2) 25m (3) 25.5m (4) 30m
 
Soln. (1) If we take east as x-axis and north as y-axis, then displacement  20 i 10 j
So, magnitude of displacement a  202 10 2 10 5  22.5m

Corporate Office : B-58, Goal Building, Budha Colony, Patna-1, Help line : 9334594165/66/67 Website : www.goalinstitute.org
Kinematics [ 5 ]
Ex.4. A body moves over one fourth of a circular arc in a circle of radius r. The magnitude of distance
travelled and displacement will be respectively
r r r
(1) ,r 2 (2) ,r (3) r, (4) r, r
2 4 2
 
Soln. (1) Let particle starts from A, its position vector rOA r i
  y
After one quarter position vector rOB r j . B
 
So displacement  r j  r i
O X
A
Magnitude of displacement r 2.

2r r
and distance = one fourth of circumference  
4 2
Ex.5. The displacement of the point of the wheel initially in contact with the ground, when the wheel
rolls forward half a revolution will be (radius of the wheel is R)
R
(1) (2) R 2  4 (3) 2R (4) R
2  4
Soln. (2) Horizontal distance covered by the wheel in half revolution = R
So the displacement of the point which was initially in contact with a ground
Pnew

2R

Pinitial
R

 (R)2  (2R)2

 R (   4).
4.5 SPEED AND VELOCITY :
(1) Average speed and Average velocity :
Rate of distance covered with time interval is called average speed.
S Total distance travelled
Average speed Vav   t  total time
(i) Time Average Speed : When particle moves with different uniform speed v1, v 2 and v3...... in
different time interval t1, t2 and t3 ....... respectively
v 1t 1  v 2t 2  v 3t 3 ....
Vav 
t1  t 2  t 3 ....
(ii) Distance Average Speed : It is defined as
d1  d2  d 3  ....
Vav 
d1  d2  d3 ....
v1 v2 v3
where d1, d2 and d3 .... etc are the distance travelled with speed v1, v 2 and v 3.

Corporate Office : B-58, Goal Building, Budha Colony, Patna-1, Help line : 9334594165/66/67 Website : www.goalinstitute.org
Kinematics [6]

(iii) Average Velocity : It is defined as the ratio of displacement to time taken by body.

  r net displacement
vav  
t total time
 
 r2  r1
vav 
t 2 t1
IMPORTANT POINTS – 2
(a) Average speed is scalar while average velocity is a vector, both having same units
(m/s) and dimensions (LT–1 )
(b) Average speed or velocity depends on time interval over which it is defined.
(c) For a given time interval average velocity is single valued while average speed can
have many values depending on path followed.
  
(d) If after motion body comes back to its initial position then v ab  0 (as  r  0) but
v av > 0 and finite as (s > 0).
 
 r  dr
(e) v  lim v for instantaneous velocity
t  t dt
(f) For a moving body average speed can never be negative or zero (unless t ) while

average velocity can be i.e. vav > 0 while v a v  0,  0 or  0.
(2) Instantaneous Speed and instantaneous Velocity : It is the speed of a particle at particular
instant
s ds
V  lim  V
t
t 0 dt
instantaneous velocity is defined as
 
 r  dr
v  lim  .V
t 0 t dt
IMPORTANT POINTS – 3
(a) Instantaneous velocity is always tangential to the path followed by the particle. When a
stone is thrown from point O then at point of projection the instantaneous velocity of
stone is v 1, at point A the instantaneous velocity of stone is v2, similarly at point B and
C are v 3 and v 4 respectively.

v3
v2
B v4
A
C
v1
O X

Direction of these velocities can be found out by drawing


a tangent on the trajectory at a given point.
Corporate Office : B-58, Goal Building, Budha Colony, Patna-1, Help line : 9334594165/66/67 Website : www.goalinstitute.org
Kinematics [7]

(b) A particle may have constant instantaneous speed but variable instantaneous velocity.
Example : When a particle is performing uniform circular motion then for every instant
of its circular motion its speed remains constant but velocity changes at every instant.
(c) The magnitude of instantaneous velocity is equal to the instantaneous speed.
(d) If a particle is moving with constant velocity then its average velocity and instantaneous
velocity are always equal.
(e) If displacement is given as a function of time, then time derivative of displacement will

give velocity. Let displacement x  A0  A1t  A 2t 

 dx d
Instantaneous velocity v   (A0  A1t  A 2t 2 )
dt dt

v  A1  2A 2t
For the given value of t, we can find out the instantaneous velocity.
 
e.g. for t = 0, Instantaneous velocity v  –A1 and Instantaneous speed v  A1

Solved Example – 2
Ex.6. If a car covers 2/5th of the total distance with v1 speed and 3/5th distance with v2 then average
speed is
1 v1  v2 2v 1v 2 5v 1v 2
(1) vv (2) (3) (4) 3v  2v
2 1 2 2 v1  v2 1 2

Total distance travelled x


Soln. (4) Average speed = 
Total time taken t1  t 2

(2/5)x (3/5)x

t1 t2

x 5v 1v 2

= (2 /5)x (3/ 5)x 2v 2  3v 1

v1 v2
Ex.7. A car accelerated from initial position and then returned at initial point, then
(1) Velocity is zero but speed increases (2) Speed is zero but velocity increases
(3) Both speed and velocity increase (4) Both speed and velocity decrease
Soln. (1) As the net displacement = 0
Hence velocity = 0 ; but speed increases.

Average velocity
Note : Average speed
 1  Average speed  Averag velocity

Corporate Office : B-58, Goal Building, Budha Colony, Patna-1, Help line : 9334594165/66/67 Website : www.goalinstitute.org
Kinematics [ 8 ]
Ex.8. A man walks on a straight road from his home to a market 2.5 km away with a speed of 5 km/h.
Finding the market closed, he instantly turns and walks back home with a speed of 7.5 km/h.
The average speed of the man over the interval of time 0 to 40 min. is equal to

25 30 45
(1) 5 km/h (2) km/h (3) km/h (4) km/h
4 4 8
2.5 1
Soln. (4) Time taken in going to market   hr  30min.
5 2
As we are told to find average speed for the interval 40 min., so remaining time for consideration of
motion is 10 min.

So distance travelled in remaining 10 min =  7.5 10  1.25km.


60
Totaldistance (2.5 1.25)km
Hence, average speed  
Total time (40/60)hr
45
 km/hr.
8
Ex.9. The motion of a particle is described by the equation x = a + bt2 where a = 15 cm and b= 3cm.
Its instantaneous velocity at time 3 sec will be
(1) 36 cm/sec (2) 18 cm/sec (3) 16 cm/sec (4) 32 cm/sec
Soln. (2) x = a + bt 2

dx
 v  0  2bt
dt
At t = 3 sec, v = 2 × 3 × 3 = 18cm/sec (As b = 3cm)
Ex.10. A train has a speed of 60 km/h for the first one hour and 40 km/h for the next half hour. Its
average speed in km/h is
(1) 50 (2) 53.33 (3) 48 (4) 70
1 3
Soln. (2) Total distance travelled  60×1 40× 1  80 km and Total time taken  1hr  hr  hr
2 2 2
80
 Average speed  3/2  53.33 km/h
Ex.11. A person completes half of its his journey with speed v1 and rest half with speed v2. The average
speed of the person is
v1  v 2 2v1v 2 v 1v 2
(1) v (2) v  v  v (3) v  v  v (4) v  v 1  v 2
2 1 2 1 2

Soln. (2) In this problem total distance is divided into two equal parts. So
dd
d1  d2
v av   2 2
d1 d2 d/ 2 d/2
 
v1 v 2 v 1 v 2
2 2v v
v av   1 2
 1  1 v1  v2
v1 v 2

Corporate Office : B-58, Goal Building, Budha Colony, Patna-1, Help line : 9334594165/66/67 Website : www.goalinstitute.org
Kinematics [9]

4.6 ACCELERATION : Acceleration is a vector quantity, defined as the rate of change of velocity.
Possible way by which change in velocity may occur.
When only direction of velocity When only magnitude of When both magnitude and
changes velocity changes direction of velocity changes
Accelerationis perpendicular to Acceleration parallel or anti-parallel Acceleration has two components
velocity to velocity one is perpendicular to velocity and
another parallel or anti-parallel to
velocity
e.g. Uniform circular motion e.g. Motion under gravity e.g. Projectile motion

  

(1) Average acceleration : Average acceleration is given by a av  v  v 2  v 1
t t
The direction of average acceleration vector is the direction of the change in velocity vector.


(2) Instantaneous acceleration : It is defined as a  lim v  dv
t 0 t dt
Ex. If a particle is accelerated for a time t1 by acceleration a1 and for time t2 by acceleration a2 then
find average acceleration.
Soln. If initial velocity is u then final velocity (after time t1 + t2) is
v = u + a1t1 + a2t2
u a1t1  a 2t 2 u a1t 1  a 2 t2
By definition a av  
t1  t 2 t1  t 2
Ex. If same force is applied on two different bodies m1 and m2 separately then it produce acceleration
a1 and a2 respectively. Now these bodies are attached together and same force is applied then
find the acceleration of confined system of m1 and m2.

Soln. F F F
a1  , a2  and a  ,
m1 m2 m1  m 2

F F F
m1  m2   and m1  m 2 
a1 a 2 a
now we have
a a F
F  1 2  
 a1a 2  a

a1a 2
 a
a1 a2

Corporate Office : B-58, Goal Building, Budha Colony, Patna-1, Help line : 9334594165/66/67 Website : www.goalinstitute.org
Kinematics [ 10 ]

IMPORTANT POINTS – 4
(a) To find the direction of average acceleration use vector mathematics, the direction of change

in velocity  V  vector gives the direction of average acceleration (not acceleration).
(b) A body is said to have uniform acceleration if magnitude and direction of the acceleration
remains constant.
(c) A body is said to have non-uniform acceleration if the magnitude or direction or both, change
during motion.
(d) For a moving body instantaneous acceleration independent from direction of instantaneous
velocity.
Ex.
(i) Uniform circular motion
(ii) Projectile
(e) Positive acceleration means velocity increasing with time

Solved Example – 3
Ex.12. A particle is moving eastwards with velocity of 5 m/s. In 10 sec the velocity changes to 5 m/s
northwards. The average acceleration in this time is
1
(1) Zero (2) m/ s2 toward north-west
2
(3) toward north-east (4) toward north-west
Soln. (2)

1
–v

 
v2 =5m/s
v
90º

v1 =5m/s

  
 v  v2  v1
v  v12  v 22  2v 1v 2 cos 90o  52  5 2  5 2
v  5 2
v 5 2 1
Average acceleration    m/s 2 toward north-west (As clear from the figure).
t 10 2
Ex.13. If displacement of a particle is directly proportional to the square of time. Then particle is moving
with
(1) Uniform acceleration (2) Variable acceleration
(3) Uniform velocity (4) Variable acceleration but uniform velocity

Corporate Office : B-58, Goal Building, Budha Colony, Patna-1, Help line : 9334594165/66/67 Website : www.goalinstitute.org
Kinematics [ 11 ]
Soln. (1) Given that x  t2 or x = Kt2 (where K= constant)
dx dv
Velocity (v)   2Kt and Acceleration (a)   2k
dt dt
It is clear that velocity is time dependent and acceleration does not depend on time.
So we can say that particle is moving with uniform acceleration but variable velocity.
Ex.14. A body of mass 10 kg is moving with a constant velocity of 10 m/s. When a constant force acts
for 4 sec on it, it moves with a velocity 2 m/sec in the opposite direction. The acceleration
produced in it is
(1) 3m/s 2 (2) – 3m/s 2 (3) 0.3 m/s 2 (4) – 0.3 m/s 2
Soln. (2) Let particle moves towards east and by the application of constant force it moves towards
west
 
v 1  10m/s and v 2  2m/s .

 
Change in velocity v 2  v 1
Acceleration  
Time t

(2) (10) 12


 a  3 m/s 2
4 4

Note

Corporate Office : B-58, Goal Building, Budha Colony, Patna-1, Help line : 9334594165/66/67 Website : www.goalinstitute.org
Kinematics [ 12 ]

Daily Practice Paper - 1


1. A body moves 6 m north. 8 m east and 10m vertically upwards, what is its resultant displacement
from initial position
10
(1) 10 2m (2) 10m m
(3) (4) 10 × 2m
2
2. If the displacement of a particle is zero, then what can we say about its distance covered
(1) It must be zero (2) It cannot be zero
(3) It is negative (4) It may or may not be zero
3. The location of a particle has changed. What can we say about the displacement and the distance
covered by the particle
(1) Both cannot be zero
(2) One of the two may be zero
(3) Both must be zero
(4) If one is positive, the other is negative and vice versa
4. The ratio of the numerical values of the average velocity and average speed of a body is always
(1) Unity (2) Unity or less (3) Unity or more (4) Less than unity
5. A particle moves along a semicircle of radius 10m in 5 seconds. The velocity of the particle is
(1) 2ms –1 (2) 4ms –1 (3) 2ms –1 (4) 4ms –1
6. A car moves for half of its time at 80 km/h for rest half of time at 40 km/h. Total distance covered
is 60 km. What is the average speed of the car
(1) 60 km/h (2) 80 km/h (3) 120 km/h (4) 180 km/h
7. A car travels a distance of 2000 m. If the first half distance is covered at 40 km/h and the second
half with speed v and the average speed is 48 km/h then the value of v is
(1) 56 km/h (2) 60 km/h (3) 50 km/h (4) 48 km/h
8. If the body covers one-third distance at speed v1, next one third at speed v2 and last one third at
speed v3, then average speed will be

v1 v2  v 2 v3  v 3 v1 v1  v2  v3
(1) v1  v2  v3 (2)
3

v1 v 2 v 3 3v1 v 2 v 3
(3) v1 v2  v 2 v3  v 3 v1 (4) v1 v2  v 2 v3  v 3 v1
9. A body moves with constant speed 10 m/s in x-axis and takes left turn then average acceleration if
time is 2 sec.
(1) 5 2m/ s2 (2) 10 2m/s 2 (3) 5 m/s 2 (4) 10m/s 2
10. If acceleration of mass 3 kg is a1 under force then same force produce acceleration a2 on 6 kg mass.
If same force is a applied on mass 9 kg then acceleration a is
(1) 3 m/s 2 (2) 2m/s 2 (3) 4m/s 2 (4) 1m/s 2

Corporate Office : B-58, Goal Building, Budha Colony, Patna-1, Help line : 9334594165/66/67 Website : www.goalinstitute.org
Kinematics [ 13 ]

11. If acceleration and velocity are in same direction then


(1) velocity increases (2) velocity decreases
(3) only direction change (4) velocity remains unchanged
12. If a particle is accelerated for a time 2 sec by acceleration 2m/s2 and for time 3sec with acceleration
7m/s 2 then average acceleration is
(1) 4m/s 2 (2) 5m/s 2 (3) 2m/s 2 (4) 10 m/s 2

Answer Daily Practice Paper - 1


1. (1) 2. (4) 3. (1) 4. (2) 5. (4)
6. (1) 7. (2) 8. (4) 9. (1) 10. (2)
11. (1) 12. (2)

4.7 EQUATIONS OF KINEMATICS.


(1) When particle moves with zero acceleration.
(i) v = u and (ii) s = ut
(2) When particle moves with constant acceleration
  
(i) v  u a t
  1    1
(ii) s  u t  a t 2 or s (t)  s  u t  a t2
2 2
  
(iii) v · v – u · u 2 a · s
Note : Equation (i) and (ii) have vector form but (iii) has scalar form.
Ex. A body starts from rest and moves with uniform acceleration then distance covered by the body in
t sec is proportional to
1
(1) t (2) (3) t2 (4) t0
t
Ans. (3)
Ex. A body starts from rest under uniform acceleration. Finds the ratio of distance covered in 1 sec,
2 sec and 3 sec.
1 1 1
Soln. s1  a(1)2, s 2  a(2)2 , s 3  a(3)2
2 2 2
s 1 : s 2 : s 3 : : 12 : 22 : 32 or 1 : 4 : 9
a
Ex. Prove that the relation Snth  u  (2n 1) where Snth is the distance travelled in nth sec.
2
n
Sol . Distance travelled by body in n sec
a
Sn = un  n2 …(i)
2
and distance travelled by body in (n – 1)sec
a
Sn – 1 = u(n – 1) + (n 1)2 …(ii)
2
Corporate Office : B-58, Goal Building, Budha Colony, Patna-1, Help line : 9334594165/66/67 Website : www.goalinstitute.org
Kinematics [ 14 ]

from (i) and (ii)


Distance travelled in nth sec

Snth  u  a (2n 1)


2
Ex. A body moving with a velocity u is stopped by application of brakes after covering a distance S1. If
the same body moves with velocity nu and same breaking force is applied on it then prove that it
will come to rest after covering the distance proportional to n2.
Soln. Using, v 2 = u2 – 2as
we have 0 = u2 – 2as 1 …(i)
and 0 = n2u2 – 2as 2 …(ii)

s2 n2 s1

Solved Example – 4
Ex.15. A body A moves with a uniform acceleration and zero initial velocity. Another body B, starts
from the same point moves in the same direction with a constant velocity . The two bodies meet
after a time t. The value of t is

2v v v v
(1) (2) (3) (4)
a a 2a 2a

1
Soln. (1) Let they meet after time ‘t’ . Distance covered by body A =  at2 ; Distance covered by
2
body B = vt
1 2  2v
and at  vt t .
2 a
Ex.16. A car, moving with a speed of 50 km/hr, can be stopped by brakes after at least 6m. If the same
car is moving at a speed of 100 km/hr, the minimum stopping distance is
(1) 6m (2) 12m (3) 18m (4) 24m
Soln. (4) v 2  u2  2as

 0  u2  2as

u2
s   s  u2 (As a = constant)
2a
Ex.17. The velocity of a bullet is reduced from 200m/s to 100m/s while travelling through a wooden
block of thickness 10cm. The retardation, assuming it to be uniform, will be
(1) 1010 4 m/s 2 (2) 1210 4 m/s 2 (3) 13.5104 m/ s 2 (4) 1510 4 m/s 2
Soln. (4) u  200m /s,v 100m/s,s  0.1m

u2  v 2 (200)2 (100)2
a 
2s 2× 0.1

 –15×10 4 m/s 2
Corporate Office : B-58, Goal Building, Budha Colony, Patna-1, Help line : 9334594165/66/67 Website : www.goalinstitute.org
Kinematics [ 15 ]
Ex.18. The average velocity of a body moving with uniform acceleration travelling a distance of 3.06 m is
0.34 ms–1. If the change in velocity of the body is 0.18ms–1 during this time, its uniform acceleration
is
(1) 0.01 ms –2 (2) 0.02 ms –2 (3) 0.03 ms –2 (4) 0.04 ms –2
Distance
Soln. (2) Time 
Average velocity
3.06
  9sec
0.34
Change in velocity
and Acceleration 
Time
0.18
 0.02m/s 2
9
Note : Can be solved by method of average acceleration
Ex.19. A particle travels 10m in first 5 sec and 10m in next 3 sec. Assuming constant acceleration, what
is the distance travelled in next 2 sec
(1) 8.3 m (2) 9.3 m (3) 10.3 m (4) None of above
Soln. (1) Let initial (t = 0) velocity of particle = u
1
for first 5 sec of motion s5 = 10 metre so by using s  ut  at 2
2
1
10  5u  a(5)2
2

 2u  5a  4 …. (i)

for first 8 sec of motion s8 = 20 m


1
20  8u  a(8)2
2

 2u  8a  5 .... (ii)

7 1
By solving (i) and (ii) u  m /s a  m/ s 2
6 3

Now distance travelled by particle in total 10 sec.

1 2
s10  u10  a10
2
by substituting the value of u and a we will get s10  28.3 m
So the distance in last 2 sec = s10 – s 8
 28.3  20  8.3 m

Corporate Office : B-58, Goal Building, Budha Colony, Patna-1, Help line : 9334594165/66/67 Website : www.goalinstitute.org
Kinematics [ 16 ]

Ex.20. A body travels for 15 sec starting from rest with constant acceleration. If it travels distances in
the first five seconds, second five seconds and next five seconds are S1, S2 and S3 respectively
the relation between S1, S2 and S3 is
1 1
(1) S1  S2  S3 (2) 5S1  3S2  S3(3) S1  1 S2  1S3 (4) S1  S2  S3
3 5 5 3
Soln. (3) Since the body starts from rest. Therefore u = 0.
1 25a
S1  a(5)2 
2 2

1 100a
S1  S2  a(10)2 
2 2

100a a
 S2   S1  75
2 2

1 225a
S1  S2  S3  a(15)2 
2 2

225a 125a
 S3   S2 S1 
2 2

Thus Clearly S1  1 S2  1S3


3 5
Ex.21. If a body having initial velocity zero is moving with uniform acceleration 8m/s2 the distance
travelled by it in fifth second will be
(1) 36 metres (2) 40 metres (3) 100 metres (4) Zero
1
Soln. (1) Sn  u  a(2n 1)
2
1
 0  (8)[2 5  1]  36 metres
2
Ex.22. A body starts from rest. What is the ratio of the distance travelled by the body during the 4th and
3rd second.
(1) 7/5 (2) 5/7 (3) 7/3 (4) 3/7

S4 7
Soln. (1) As Sn (2n 1), 
S3 5
4.8 MOTION UNDER GRAVITY :
In the absence of the air resistance, all bodies falls with the same acceleration near the surface
of the earth. The motion of body called free fall. For motion under gravity a = g m/s2 or a = 32 ft s 2
When a body dropped from some height its initial velocity equal to zero and
1
h  gt 2
2
v 2 = 2gh

Corporate Office : B-58, Goal Building, Budha Colony, Patna-1, Help line : 9334594165/66/67 Website : www.goalinstitute.org
Kinematics [ 17 ]

If a body is projected vertically If a body is projected vertically


downward with some initial velocity upward with some initial velocity

(1) v = u + gt (1) v = u – gt
1 2 1 2
(2) h = ut + gt (2) h = ut – gt
2 2
(3) v 2 = u2 + 2gh (3) v 2 = u2 – 2gh
g g
(4) hn  u  (2n 1) (4) hn  u  (2n 1)
2 2

Different Graphs for Motion Under Gravity When a = +g

s v a

g
tan = g

t t t

IMPORTANT POINTS–5
(a) As h = (1/2)gt2, i.e., h  t2, distance covered in time t, 2t, 3t, etc., will be in the ratio of
12 : 22 : 32, i.e., square of integers.
1
(b) The distance covered in the nth sec, hn  g (2n 1) if body starts fromrest
2
(c) In case of motion under gravity for a given body, mass, acceleration, and mechanical
energy remain constant while speed, velocity, momentum, kinetic energy and potential
energy change.
(d) The motion is independent of the mass of the body, as in any equation of motion, mass
is not involved. That is why a heavy and light body when released from the same height,
reach the ground simultaneously and with same velocity i.e, t  (2h/g) and v  2gh.
(e) In case of motion under gravity time taken to go up is equal to the time taken to fall down
through the same distance. Time of descent (t1) = time of ascent (t2) = u/g
(f) In case of motion under gravity, the speed with which a body is projected up is equal to
the speed with which it comes back to the point of projection.
(g) A body is thrown vertically upwards. If air resistance is to be taken into account, then the
time of ascent is less than the time of descent. t2 > t1
(h) A particle is dropped vertically from rest from a height. The time taken by it to fall through
successive distance of 1m each will then be in the ratio of the difference in the square
roots of the integers i.e.
1,( 2 1),  3  2( 4  3),
(i) Time taken by an observer to comes in action when need it called reaction time.

Corporate Office : B-58, Goal Building, Budha Colony, Patna-1, Help line : 9334594165/66/67 Website : www.goalinstitute.org
Kinematics [ 18 ]
Ex. Under the effect of air resistance time of ascent is less than the time of descent t2 > t1 proved.
u u2
Soln. Let u is the initial velocity of body then time of ascent t 1  g  a and h  2(g  a)
where g is acceleration due to gravity and a is retardation by air resistance and for upward motion
both will work vertically downward.
For downward motion a and g will work in opposite direction because acceleration due to air effect
always work in direction opposite to motion and g always work vertically downward.
1
So h  (g  a)t 22
2
u2 1 2
 2(g  a)  2(g  a)t 2

u
 t2 
(g  a)(g  a)
Comparing t1 and t2 we can say that t2 > t1 since (g + a ) > (g – a)

Solved Example – 5
Ex.23. If a body is thrown up with the velocity of 15 m/s then maximum height attained by the body is
(g = 10 m/s2)
(1) 11.25 m (2) 16.2 m (3) 24.5 m (4) 7.62 m

Soln. (1) u2 (15)2


Hmax    11.25m
2g 210
Ex.24. A body falls from rest in the gravitational field of the earth. The distance travelled in the
5th second of its motion is
(1) 25m (2) 45m (3) 90m (4) 125m
g
Soln. (2) hn  2n 1
2
10

2
25 1  45m.
h5 th 

Ex.25. If a ball is thrown vertically upwards with speed u, the distance covered during the last t seconds
of its ascent is
1 2 1
(1) gt (2) ut  gt 2 (3) (u  gt)t (4) ut
2 2
u
Soln. (1) If ball is thrown with velocity u, then time of flight  g

u  u 
velocity after  g  t sec: v  u  g g  t   gt. h t sec
   
So, distance in last ‘t’ sec : 02 = (gt)2 –2(g)h. u
sec t
1 g
 h  gt2 .
2

Corporate Office : B-58, Goal Building, Budha Colony, Patna-1, Help line : 9334594165/66/67 Website : www.goalinstitute.org
Kinematics [ 19 ]

Ex.26. A man throws balls with the same speed vertically upwards one after the other at an interval
of 2 seconds. What should be the speed of the throw so that more than two balls are in the
sky at any time Given :
(1) At least 0.8 m/s (2) Any speed less than 19.6 m/s
(3) Only with speed 19.6 m/s (4) More than 19.6 m/s
Soln. (4) Interval of ball throw = 2 sec.
If we want that minimum three (more than two) ball remain in air then time of flight of first
ball must be greater than 4 sec.
2U
i.e. T  4sec or  4sec  u  19.6m /s
g
It is clear that for first ball will just strike the ground (in sky), second ball will be at highest
point (in sky), and third ball will be at point of projection or on ground (not in sky).
Ex.27. A very large number of balls are thrown vertically upwards in quick succession in such a way
that the next ball is thrown when the previous one is at the maximum height. If the maximum
height is 5m, the number of ball thrown per minute is (take g = 10ms–2 )
(1) 120 (2) 80 (3) 60 (4) 40
Soln. (3) Maximum height of ball = 5m, So velocity of projection
 u  2gh  210 5 10m/s

u 1
time interval between two balls (time of ascent)  g 1sec  60 min.

So no. of ball thrown per min = 60


Ex.28. A particle is thrown vertically upwards. If its velocity at half of the maximum height is 10 m/s,
then maximum height attained by it is (Take m/s2)
(1) 8 m (2) 10 m (3) 12 m (4) 16 m
2
Soln. (2) Let particle thrown with velocity u and its maximum height is H then H  u
2g
When particle is at a height H/2, then its speed is 10m/s
From equation , v 2  u2  2gH,

H u2
 10  u2  2g    u2  2g
2
 u2  200
2 4g

 u2  200

u2 200
 Maximum height H  2g  210 10m

Ex.29. A body freely falling from the rest has a velocity ‘v’ after it falls through a height ‘h’. The distance
it has to fall down for its velocity to become double, is
(1) 2h (2) 4h (3) 6h (4) 8h

Corporate Office : B-58, Goal Building, Budha Colony, Patna-1, Help line : 9334594165/66/67 Website : www.goalinstitute.org
Kinematics [ 20 ]
Soln. (2) Let at point A initial velocity of body is equal to zero
For path AB : v2 = 0 + 2gh … (i)
For path AC : (2v)2  0  2gx
 4v 2  2gx … (ii)
Solving (i) and (ii) x = 4h
Ex.30. A stone dropped from a building of height h and it reaches after t seconds on earth. From the
same building if two stones are thrown (one upwards and other downwards) with the same
velocity u and they reach the earth surface after and seconds respectively, then
t 1  t2
(1) t  t1  t2 (2) t (3) t  t1t 2 (4) t  t21 t 22
2
1
Soln. (3) For first case of dropping h  gt 2.
2

1 1
For second case of upward throwing h  ut1  gt12  gt2
2 2

1
 ut1  g(t 2  t12 ) ......(i)
2
1 1
For third case of downward throwing h  ut 2  gt 22  gt2
2 2
1
 ut 2  g(t 2  t 22 ) .......(ii)
2
t1 t 2  t12
on solving these two equations :  t  t 2  t 2  t  t1t 2 .
2 2

Ex.31. Water drops fall at regular intervals from a tap which is 5 m above the ground. The third drop is
leaving the tap at the instant the first drop touches the ground. How far above the ground is the
second drop at that instant
(1) 2.50 m (2) 3.75 m (3) 4.00 m (4) 1.25 m
Soln. (2) Let the interval be t then from question

1
For first drop g(2t)2  5 …(i)
2

1
For second drop x  gt 2 …(ii)
2

5 5
By solving (i) and (ii) x  and hence required height h  5   3.75m.
4 4
Ex.32. A balloon is at a height of 81 m and is ascending upwards with a velocity of 12 m/s. A body of
2 kg weight is dropped from it. The body will reach the surface of the earth in
(1) 1.5 s (2) 4.025 s (3) 5.4 s (4) 6.75 s
Soln. (3) As the ballo on is going up we will t ake initial velocity of falling body
 12m /s, h  81m, g  10m/ s 2

Corporate Office : B-58, Goal Building, Budha Colony, Patna-1, Help line : 9334594165/66/67 Website : www.goalinstitute.org
Kinematics [ 21 ]

1 1
By applying ; h  ut  gt2 ; 81 12t  (10)t 2
2 2
 5t2 12t  81 0

12  144 1620
 t
10

12  1764
  5.4sec.
10
Ex.33. A particle is dropped under gravity from rest from a height h and it travels a distance 9h/25 in the
last second, the height h is
(1) 100 m (2) 122.5 m (3) 145 m (4) 167.5 m
1
Soln. (2) Distance travelled in nsec  gn2  h …(i)
2
g 9h
Distance travelled in nth sec  (2n 1)  …(ii)
2 25
Solving (i) and (ii) we get h = 122.5m.

Note

Corporate Office : B-58, Goal Building, Budha Colony, Patna-1, Help line : 9334594165/66/67 Website : www.goalinstitute.org
Kinematics [ 22 ]

Daily Practice Paper - 2


1. A body of 5 kg is moving with a velocity of 20 m/s. If a force of 100 N is applied on it for 10s in the
same direction as its velocity, what will now be the velocity of the body
(1) 200 m/s (2) 220 m/s (3) 240 m/s (4) 260 m/s
2. A particle is constrained to move on a straight line path. It returns to the starting point after 10 sec.
The total distance covered by the particle during this time is 30 m. Which of the following statements
about the motion of the particle is false
(1) Displacement of the particle is zero (2) Average speed of the particle is 3 m/s
(3) Displacement of the particle is 30 m (4) Both (1) and (2)
3. If a body starts from rest and travels 120 cm in the 6th second, then what is the acceleration
(1) 0.20 ms–2 (2) 0.027 ms–2 (3) 0.218 ms–2 (4) 0.03 ms–2
4. A particle moving with a uniform acceleration travels 24 m and 64 m in the first two consecutive
intervals of 4 sec each. Its initial velocity is
(1) 1 m/sec (2) 10 m/sec (3) 5 m/sec (4) 2 m/sec
5. A particle starts from rest and moving with constant acceleration covers a distance s1 in the
3rd second and s2 in the 5th second. The ratio s1 and s 2
(1) 3/5 (2) 5/9 (3) 9/25 (4) 25/81
6. A bullet moving with a velocity of 200 cm/s penetrates a wooden block and comes to rest after
traversing 4 cm inside it. What velocity is needed for travelling distance of 9 cm in same block
(1) 100 cm/s (2) 136.2 cm/s (3) 300 cm/s (4) 250 cm/s
7. The displacement x of a particle in time t is given by 10t2 –
4t – x = 0. Where x is in metre and t in
second. The distance covered by the body in 4th second of motion
(1) 31 m (2) 39.5 m (3) 66 m (4) 75 m
8. The speed of a body moving with uniform acceleration is u. This speed is doubled while covering
a distance S. When it covers an additional distance S, its speed would become
(1) 3u (2) 5u (3) 11u (4) 7u
9. Two cars A and B are at rest at same point initially. If A starts with uniform velocity of 40 m/sec and
B starts in the same direction with constant acceleration of 4m/s2, then B will catch A after how
much time
(1) 10 sec (2) 20 sec (3) 30 sec (4) 35 sec
10. A driver takes 0.20 s to apply the brakes after he sees a need for it. This is called the reaction time
of the driver. If he is driving a car at a speed of 54 km/h and the brakes cause a deceleration of
6.0 m/s 2, find the distance travelled by the car after he sees the need to put the brakes on.
(1) 22 m (2) 24 m (3) 26 m (4) 28 m
11. The acceleration due to gravity on the planet A is 9 times the acceleration due to gravity on planet
B. A man jumps to a height of 2 meter on the surface of A. What is the height of jump by the same
person on the planet B
2 2
(1) 18m (2) 6m (3) m (4) m
3 9

Corporate Office : B-58, Goal Building, Budha Colony, Patna-1, Help line : 9334594165/66/67 Website : www.goalinstitute.org
Kinematics [ 23 ]
12. Two balls are dropped from heights h and 2h respectively from the earth surface. The ratio of time
of these balls to reach the earth is
(1) 1: 2 (2) 2:1 (3) 2 : 1 (4) 1 : 4
13. A cricket ball is thrown up with a speed of 19.6 ms–1. The maximum height it can reach is
(1) 9.8 m (2) 19.6 m (3) 29.4 m (4) 39.2 m
14. A particle is dropped vertically from rest from a height. The time taken by it to fall through successive
distances of 1m each will then be

(1) All equal, being equal to 2/g second


(2) In the ratio of square roots of the integers 1, 2, 3, ….
(3) In the ratio of the difference in the square roots of the integers
i.e. 1,( 2  1),( 3  2),( 4  3).......
1 1 1 1
(4) , , ,
In the ratio of the reciprocal of the square roots of the integers i.e.
1 2 3 4
15. A body falling from a high Minaret travels 40 meters in the last 2 seconds of its fall to ground.
Height of Minaret in meters is (take g = 10m/s2)
(1) 60 (2) 45 (3) 80 (4) 50
16. A ball is dropped from top of a tower of 100m height. Simultaneously another ball was thrown
upward from bottom of the tower with a speed of 50 m/s ( g = 10m/s2). They will cross each other
after
(1) 1s (2) 2s (3) 3s (4) 4s
17. From the top of a tower, a particle is thrown vertically downwards with a velocity of 10 m/s. The
ratio of the distances covered by it in the 3rd and 2nd seconds of the motion is (Take g = 10 m/s2)
(1) 5: 7 (2) 7: 5 (3) 3 : 6 (4) 6 : 3
18. Three different objects of masses m1, m2 and m3 are allowed to fall from rest and from the same
point 'O' along three different frictionless paths. The speeds of the three objects, on reaching the
ground, will be in the ratio of
1 1 1
(1) m1:m2 :m3 (2) m1:2m2 :3m 3 (3) 1 : 1 : 1 (4) m : m : m
1 2 3

19. A particle when thrown, moves such that it passes from same height at 2s and 10s, the height is

(1) g (2) 2g (3) 5 g (4) 10 g

Answer Daily Practice Paper-2


1. (2) 2. (3) 3. (3) 4. (1) 5. (2)
6. (3) 7. (3) 8. (4) 9. (2) 10. (1)
11. (1) 12. (1) 13. (2) 14. (3) 15. (2)
16. (2) 17. (2) 18. (3) 19. (4)

Corporate Office : B-58, Goal Building, Budha Colony, Patna-1, Help line : 9334594165/66/67 Website : www.goalinstitute.org
Kinematics [ 24 ]
4.9 GENERAL MOTION IN A STRAIGHT LINE (NON-UNIFORM ACCELERATION) : Saying general
motion means motion is other than motion with uniform velocity or motion with uniform acceleration.
In such motion we will be given the relation between two variables among position, velocity,
acceleration and time and other are to be calculated.
For this we need to use,
dx
Velocity, v 
dt
dv vdv
Acceleration, a  
dt dx
and then using calculus we find the unknown variables.
Methods of solving questions of variable acceleration :
Case-I When acceleration a of the particle is a function of time i.e.
dv
  f(t)
dt
 dv = f(t)dt
Integrating both sides within suitable limits, we get,
v t

 dv   f(t)dt
u 0

 v u   f(t)dt
0

Case-II When acceleration of the particle is a function of distance i.e.


a = f(x)
dv
  f(x)
dt
dv dx
   f(x)
dx dt

dv
 v  f(x)
dx
 vdv  f(x)dx
Integrating, we get
v s

 vdv   f(x)dx
u 0

S
v 2  u2  f(x)dx

2 2 0

s
 v 2  u2  2  f(x)dx
0

Corporate Office : B-58, Goal Building, Budha Colony, Patna-1, Help line : 9334594165/66/67 Website : www.goalinstitute.org
Kinematics [ 25 ]
Case-III When acceleration of the particle is a function of velocity i.e. a = f(v)
dv
  f(v)
dt
Option-I Option-II
dv dv
dt  v×  f(v)
f(v) dx
Integrating, Integrating
t v v
 dv  x vdv
0 dt  u  f(v) x 0 dx  u f(v)
v v
t   dv x  x 0   vdv
u
f(v) u
f(v)
Ex. A particle moving in a straight line has an acceleration (3t –4)ms–2 at time t seconds. The particle
is initially 1m from O, a fixed point on the line, with a velocity of 2 m/s. Find the time when the
velocity is zero. Find also the displacement of the particle from O when t = 3s

Soln. Using a  dv gives


dt
dv
 3t  4
dt
v t

  dv  (3t  4)dt
2 0

3t2
 v 2   4t
2
3t 2
 v  4t  2
2
3t 2
The velocity will be zero when  4t  2  0
2
i.e. when (3t – 2)(t – 2) = 0
2
 t  or 2
3
ds ds 3t 2
Using  v,wehave   4t  2
dt dt 2
s 3

 ds 
1 0 2 
3t 2  4t  2 dt

3
3
 s 1 t  2t 2  2t  1.5
2 0
1
 s2
2
Therefore the particle is 2.5 m from O when t = 3 sec.
Corporate Office : B-58, Goal Building, Budha Colony, Patna-1, Help line : 9334594165/66/67 Website : www.goalinstitute.org
Kinematics [ 26 ]

Solved Example–6
Ex.34. An electron starting from rest has a velocity that increases linearly with the time as v = 2t m/s . The
distance travelled in the first 3 seconds will be
(1) 9m (2) 16 m (3) 27 m (4) 36 m
3 3
t2 t 2 
Soln. (1) x   t vdt  2tdt  2    9m.
1
0 2 0
Ex.35. The acceleration of a particle is increasing linearly with time t as bt. The particle starts from the
origin with an initial velocity v0. The distance travelled by the particle in time t will be
1 1 1 1
(1) v 0t  bt 2 (2) v 0t  bt 3 (3) v 0t  bt 3 (4) v 0t  bt 2
3 3 6 2
v t

Soln. (3)  dv   adt


v0 0

v t

  dt  (bt)dt.
v0 0

 v  v 0  bt
2
2
  0

bt 2
 v  v0 
2
dx bt 2
  v0 
dt 2
s t

 
bt
0 dx  0 v 0  2 dt
2


bt 3
 s  v0 t 
6
Ex.36. The motion of a particle is described by the equation u = at. The distance travelled by the
particle in the first 4 seconds
(1) 4a (2) 12a (3) 6a (4) 8a

Soln. (4) u  at

ds
  at
dt
4 4
t 2 
 s  atdt  a  
0 2 0

 8a.

Corporate Office : B-58, Goal Building, Budha Colony, Patna-1, Help line : 9334594165/66/67 Website : www.goalinstitute.org
Kinematics [ 27 ]
4.10 Analysis of motion through graph :
(1) Displacement-time graph :
If displacement of a body is plotted on Y-axis and time on X-axis, the curve obtained is called
displacement-time graph. The instantaneous velocity at any given instant can be obtained from
the graph by finding the slope of the tangent at the point corresponding to the time.

a v=0 b
e
Displacement

ng g
as i ty c d
re ci t v-constant
e c lo an
d ve st
c on
v= f
increasing
velocity
O t0 x
Time

If the graph obtained is parallel to time axis, the velocity is zero (ab in figure). If the graph is an
oblique straight line, the velocity is constant (oc and ef in figure). If the graph obtained is a curve
(od in figure), whose slope increases with time, the velocity goes on increasing i.e. the motion is
accelerated. If the graph obtained is a curve of type og in figure whose slope decreases with time,
the velocity goes on decreasing.
(2) Velocity time graph : Similarly a graph can be drawn between velocity and time of a moving
body. The curve obtained will be similar to the one shown in figure. If the graph is a straight line
parallel to time axis, the velocity is constant and acceleration is zero (AB in figure). If the graph is
an oblique straight line, the motion is uniformly accelerated (positive slope) or uniformly decelerated
(negative slope). The velocity-time curve may be used to determine displacement, velocity and
acceleration i.e. it is used to specify the entire motion.

velocity constant (a=0)


A B
g g
sin c d
r ea v-constant
Velocity

de
c
ant d
a t
ns
co
a a increasing

Time (t) e x

Corporate Office : B-58, Goal Building, Budha Colony, Patna-1, Help line : 9334594165/66/67 Website : www.goalinstitute.org
Kinematics [ 28 ]
To obtain the velocity as any time t, we draw a perpendicular from
given instant on the curve and noting the corresponding point and y
the dropping a perpendicular from this point on the velocity axis.
The slope of the tangent at the point corresponding to the particular q
time on the curve gives instantaneous acceleration.

Velocity (v)
The area enclosed by velocity–time graph and time axis for a time
interval gives the displacement during this time interval. p

In figure the area pqt2t1 represents the net displacement during


x
the time interval between t1 and t2. t1 t2

(3) Acceleration-time graph : It is a graph plotted between time and acceleration. If the graph is a
line parallel to time axis, the acceleration is constant. If it is a straight line with positive slope, the
acceleration is uniformly increasing. The co-relation of the graph explained above follows directly
dS dv
from the differential expression, v  and a 
dt dt
a
non-uniform
acceleration

uniform
acceleration
non-uniform
acceleration

t a
uniform
velocity
q
The area enclosed between acceleration–time graph
p
and time axis during this time interval gives change
in velocity. t1 t2 t

Ex. At t = 0 a particle is at rest at origin. Its acceleration is 2 ms–2 for the first 3s and – 2ms–2 for the
next 3s. Find the acceleration versus time, velocity versus time and position versus time graph.
Soln. We are given that for first 3s acceleration is 2ms–2 and for next 3s acceleration is –2ms–2. Hence
acceleration time graph is as show in the figure.

Y
2
a(m/s )
2
2m/s
O
3 3 t(s)X

–2m/s2

The area enclosed between a – t curve gives change in velocity for the corresponding interval.
Also at t = 0, v = 0, hence final velocity at t = 3s will increase to 6ms–1. In next 3s the velocity will
decrease to zero. Hence the velocity time graph is as shown in figure.
Corporate Office : B-58, Goal Building, Budha Colony, Patna-1, Help line : 9334594165/66/67 Website : www.goalinstitute.org
Kinematics [ 29 ]

V(m/s)

0 X
3 6 t(s)

Note : v – t curves are taken as straight line as acceleration is constant.


Now for displacement time curve, we will use the fact that area enclosed between v - t curve gives
displacement for the corresponding interval. Hence displacement in first three seconds is 4.5 m
and in next three seconds is 4.5 m. Also the x – t curve will be of parabolic nature as motion is with
constant acceleration. Therefore, x – t curve is as shown in figure.

Y
x(m)

4.5

X
0 3 6 t(s)

IMPORTANT POINTS–6

(a) For uniformly accelerated motion (a 0), x – t graph is a parabola opening upwards if
(a > 0) and opening downwards if (a < 0). The slope of tangent at any point of the parabola
gives the velocity at that instant.
(b) For uniformly accelerated motion (a 0), v – t graph is straight line whose slope gives the
acceleration of the particle.
(c) In general, the slope of tangent in x – t graph is velocity and the slope of tangent in v – t graph
is the acceleration.
(d) The area under a – t graph gives the change in velocity (not velocity)
(e) The area between the v – t graph and the time-axis gives the distance travelled by the particle,
if we take all area positive.

(f) Area under v – t graph gives displacement if area below the t-axis are taken negative.

Corporate Office : B-58, Goal Building, Budha Colony, Patna-1, Help line : 9334594165/66/67 Website : www.goalinstitute.org
Kinematics [ 30 ]

Solved Example – 7
Ex.37. The position of a particle moving along the x-axis at certain times is given below :
t (s) 0 1 2 3
x (m) –2 0 6 16
Which of the following describes the motion correctly
(1) Uniform, accelerated (2) Uniform, decelerated
(3) Non-uniform, accelerated (4) There is not enough data for generalisation
x
Soln. (1) Instantaneous velocity v  , By using the data from the table
dt
0 ( 2) 61 16  6
v1   2m/s,v 2   6m/ s and v 3  10m/ s i.e. the speed is increasing at a
1 1 1
constant rate so motion is uniformly accelerated.
Ex.38. Which of the following graph represents uniform motion

s s

(1) (2)

t t
s s

(3) (4)

t t
Soln. (1) When distance time graph is a straight line with constant slope then motion is uniform.
Ex.39. The displacement-time graph for two particles A and B are straight lines inclined at angles of
30o and 60o with the time axis. The ratio of velocities of vA : v B is

(1) 1:2 (2) 1: 3 (3) 3:1 (4) 1 : 3


Soln. (4) v = tan from displacement time-graph.
v A tan30
So v  tan60
B

1/ 3 1 1
  
3 3 3 3
Ex.40. From the following displacement time graph find out the velocity of a moving body
1
(1) m/s
3
Ti me (sec)

(2) 3 m/s
(3) 3 m/s o
30
1 O
(4) Displacement (meter)
3

Corporate Office : B-58, Goal Building, Budha Colony, Patna-1, Help line : 9334594165/66/67 Website : www.goalinstitute.org
Kinematics [ 31 ]

1
Soln. (3) In first instant you will apply v = tan  and say, v  tan 30 
m/s .
3
But it is wrong because formula is valid when angle is measured with time axis.
Here angle is taken from displacement axis. So angle from time axis is 60°.
Now v  tan 60o  3
Ex.41. The diagram shows the displacement-time graph for a particle moving in a straight line. The
average velocity for the interval t = 0, t = 5 is
x
20
10
5
O t
2 4
10

(1) 0 (2) 6 ms –1 (3) – 2 ms –1 (4) 2 ms –1


Totaldisplacement
Soln. (3) Average velocity 
Total time

(20) ( 20)  (–10)


  2m/s
5
Ex.42. Figure shows the displacement time graph of a body. What is the ratio of the speed in the first
second and that in the next two seconds
m
30
Displacement

20
10
0 sec
1 2 3
Time

(1) 1: 2 (2) 1: 3 (3) 3 : 1 (4) 2 : 1


Soln. (4) Speed in first second = 30m/s and Speed in next two seconds = 15m/s. So that ratio 2 : 1
Ex.43. A ball is thrown vertically upwards. Which of the following plots represents the speed-time graph
of the ball during its flight if the air resistance is not ignored
Speed

Speed

Speed
Speed

(1) (2) (3) (4)


Time Time Time Time

Soln. (3) In first half of motion the acceleration is uniform & velocity gradually decreases, so slope
will be negative but for next half acceleration is positive. So slope will be positive. Thus
graph ‘c’ is correct.
Not ignoring air resistance means upward motion will have acceleration (a + g) and the
downward motion will have (g – a)

Corporate Office : B-58, Goal Building, Budha Colony, Patna-1, Help line : 9334594165/66/67 Website : www.goalinstitute.org
Kinematics [ 32 ]
Ex.44. A train moves from one station to another in 2 hours time. Its speed-time graph during this
motion is shown in the figure. The maximum acceleration during the journey is

100

Speed in km/hours
80

60 D
40
20 B C L

A
N M E
0.25 0.75 1.00 1.5 2.00
Time in hours

(1) 140 km/h2 (2) 160 km/h2 (3) 100 km/h2 (4) 120 km/h2
Soln. (2) Maximum acceleration means maximum slope in speed – time graph. that slope is for line
60  20
CD. So, amax  slope of CD 
1.25 1.00
40
 160km/h2 
0.25
Ex.45. The graph of displacement v/s time is

Its corresponding velocity-time graph will be

V V V
V
(1) (2) (3) (4)
t t
t t

Soln. (1) We know that the velocity of body is given by the slope of displacement – time graph. So
it is clear that initially slope of the graph is positive and after some time it becomes zero
(corresponding to the peak of the graph) and then it will be negative.
Ex.46. In the following graph, distance travelled by the body in metres is
Y
15
v (m/s)

10
5
X
0 10 20 30
Time (s)
(1) 200 (2) 250 (3) 300 (4) 400

Corporate Office : B-58, Goal Building, Budha Colony, Patna-1, Help line : 9334594165/66/67 Website : www.goalinstitute.org
Kinematics [ 33 ]
Soln. (1) Distance = The area under v – t graph
1
s  (30 10)×10  200metre
2
Ex.47. For the velocity-time graph shown in figure below the distance covered by the body in last two
seconds of its motion is what fraction of the total distance covered by it in all the seven seconds

B C
10

v (m/s )
5
A P Q D
0 1 3 5 7
Time (s)

1 1 1 2
(1) (2) (3) (4)
2 4 3 3

Soln. (2) Distance covered in total 7 seconds = Area of trapezium ABCD  1(2  6) × 10  40m
2

1
Distance covered in last 2 second = area of triangle CDQ  ×2×10  10m
2
10 1
So required fraction  
40 4
Ex.48. The velocity time graph of a body moving in a straight line is shown in the figure. The displacement
and distance travelled by the body in 6 sec are respectively

4A B
G H
v (m/s)

2
2 F 6
O C 4
–2 t (sec)
D E
–4

(1) 8m, 16m (2) 16m, 8m (3) 16m, 16m (4) 8m, 8m
Soln. (1) Area of rectangle ABCO = 4 × 2 = 8 m
Area of rectangle CDEF = 2 × (– 2) = – 4 m
Area of rectangle FGHI = 2×2 = 4 m
Displacement = sum of area with their sign = 8 + (– 4) + 4 = 8 m
Distance = sum of area without sign = 8 + 4 + 4 = 16 m
Ex.49. A ball is thrown vertically upward which of the following graph represents velocity time graph of
the ball during its flight (air resistance is neglected)
Velocity

Velocity

Velocity
Velocity

(1) (2) Time (3) Time (4) Time


Time

Corporate Office : B-58, Goal Building, Budha Colony, Patna-1, Help line : 9334594165/66/67 Website : www.goalinstitute.org
Kinematics [ 34 ]
Soln. (4) In the positive region the velocity decreases linearly (during rise) and in negative region
velocity increase linearly (during fall) and the direction is opposite to each other during rise
and fall, hence fall is shown in the negative region.
Ex.50. A ball is dropped vertically from a height d above the ground. It hits the ground and bounces up
d
vertically to a height . Neglecting subsequent motion and air resistance, its velocity v varies
2
with the height h above the ground as.

d d/2 d/2 d
(1) h (2) h
(3) h (4) h
d/2 d/2 d d

Soln. (1) When ball is dropped from height d its velocity will be zero.
As ball comes downward h decreases and v increases just before the rebound from the
earth
h = 0 and v = maximum and just after rebound velocity reduces to half and direction
becomes opposite.
d
As soon as the height increases its velocity decreases and becomes zero at h  .
2
This interpretation is clearly shown by graph (1).
Ex.51. The acceleration-time graph of a body is shown below :
a

t
The most probable velocity-time graph of the body is
v v v v

(1) (2) (3) (4)


t t t t

Soln. (3) From given a – t graph acceleration is increasing at constant rate


da
  (constant)  a = kt (by integration)
dt
dv
  kt  dv = ktdt
dt
kt 2
  dv  k  tdt 
2
v
i.e., v is dependent on time parabolically and parabola is symmetric about v-axis.
and suddenly acceleration becomes zero. i.e. velocity becomes constant.
Hence (3) is most probable graph.

Corporate Office : B-58, Goal Building, Budha Colony, Patna-1, Help line : 9334594165/66/67 Website : www.goalinstitute.org
Kinematics [ 35 ]

Ex.52. Which of the following velocity time graphs is not possible

v v v v

(1) (2) (3) (4)


O t O t O t O t

Soln. (4) Particle can not possess two velocities at a single instant so graph (4) is not possible.
Ex.53. For a certain body, the velocity-time graph is shown in the figure.
The ratio of applied forces for intervals ABand BC is
D
1 1
(1)  (2) 
2 2
60 o
1 1 30 o
t
(3)  (4)  A B C
3 3
Soln. (4) Ratio of applied force = Ratio of acceleration

a AB tan30
= 
a BC tan(120)
1/ 3
  1/ 3
3
Ex.54. Velocity-time graphs of two cars which start from rest at
the same time, are shown in the figure. Graph shows, that
Velocity

A B

(1) Initial velocity of A is greater than the initial velocity of B


A
(2) Acceleration in A is increasing at lesser rate than in B
B
(3) Acceleration in A is greater than in B
O t Time
(4) Acceleration in B is greater than in A
Soln. (3) At a certain instant t slope of A is greater than B (A > B), so acceleration in A is greater
than B
Ex.55. Which one of the following graphs represent the velocity of a steel ball which fall from a
height on to a marble floor? (Here v represents the velocity of the particle and t the time)
Velocity
Velocity
Velocity
Velocity

(1) (2) Time (3) Time


(4) Time
Time

Soln. (1) Initially when ball falls from a height its velocity is zero and goes on increasing when it
comes down. Just after rebound from the earth its velocity decreases in magnitude and its
direction gets reversed. This process is repeated until ball comes to at rest. This
interpretation is well explained in graph (1).

Corporate Office : B-58, Goal Building, Budha Colony, Patna-1, Help line : 9334594165/66/67 Website : www.goalinstitute.org
Kinematics [ 36 ]

Ex.56. The adjoining curve represents the velocity-time graph of a particle, its acceleration values
along OA, AB and BC in metre/sec2 are respectively
(1) 1, 0, – 0.5

(Velocity m/sec)
(2) 1, 0, 0.5 10 A B

(3) 1, 1, 0.5 5
(4) 1, 0.5, 0
O 10 20 30 40
Soln. (1) Acceleration along Time (sec)

v 2  v 1 10  0
OA    1m/ s 2
t 10

0
Acceleration along AB  0
10

0 10
Acceleration along BC   –0.5m/ s 2
20

Daily Practice Paper-3


1. The displacement versus time graph for a body moving in a straight line shown in figure. Which of
the following regions represents the motion when no force is acting on the body

x e
d
b c

a t

(1) ab (2) bc (3) cd (4) de


2. A car decelerates at a constant rate during a period commencing at t = 0. Which of the displacement
time graphs represents the displacement of the car
Displacement

Displacement
Displacement

Displacement

(1) (2) (3) (4)


Time Time Ti me Time

3. Which of the following can not be the distance time graph

x x x x

(1) (2) (3) (4)

t t t t

Corporate Office : B-58, Goal Building, Budha Colony, Patna-1, Help line : 9334594165/66/67 Website : www.goalinstitute.org
Kinematics [ 37 ]
4. The graph between the displacement x and time t for a particle moving in a straight line is shown
in figure. During the interval OA, AB, BC and CD, the acceleration of the particle is
OA AB BC CD
x D
(1) + 0 + +
C
(2) - 0 + 0 A B
(3) + 0 - +
(4) - 0 - 0 O t

5. The variation of velocity of a particle with time moving along a straight line is illustrated in the
following figure. The distance travelled by the particle in four seconds is
velocity (m/s)

30

20

10

0 1 2 3 4
Time in second

(1) 60 m (2) 55 m (3) 25 m (4) 30 m


6. A lift is going up. The variation in the speed of the lift is as given in the graph. What is height to
which the lift takes the passenger
Velocity (m/sec)

3.6

2 Time (sec) 10 12

(1) 3.6 m (2) 28.8 m


(3) 36.0 m (4) Cannot be calculated from the above graph
7. Which of the following velocity time graphs is possible.
v v
Velocity

Velocity

(1) (2)
t t
O Time O Time

v v
Velocit y

Velocity

(3) (4)
t t
O Time O
Time

8. A body is moving according to the equation x  at  bt 2  ct 3 where a, b and c are constants. The
acceleration of the body is
(1) a + 2bt (2) 2b + 6ct (3) 2b – 6ct (4) 3b – 6ct2
Corporate Office : B-58, Goal Building, Budha Colony, Patna-1, Help line : 9334594165/66/67 Website : www.goalinstitute.org
Kinematics [ 38 ]

9. The displacement is given by x  2t 2  t  5, the acceleration at t = 2s is


(1) 4 m/s 2 (2) 8 m/s 2 (3) 10 m/s 2 (4) 15 m/s 2
10. A particle moves along a straight line such that its displacement at any time t is given by
S = t3 – 3t2 + 2 meter. The displacement when the acceleration becomes zero is
(1) 0 meter (2) 2 meter
(3) 3 meter (4) – 2 meter
11. The x-t graph in figure represents

Displacement
(1) Constant velocity
(2) Velocity of the body continuously changing
t1
(3) Instantaneous velocity
Time (t)
(4) The body travels with constant speed upto time t1 and then stops
12. The initial velocity of a particle is u and the acceleration f is given by at. Which of the following
relation is valid

(1) v = u + at2 (2) t2 (3) v = u + at (4) v u


v ua
2
13. A particle, initially at rest, starts moving in a straight line with an acceleration a  6t  4m/s 2 . The
distance covered by it in 3 s is
(1) 30 m (2) 60 m (3) 45 m (4) 15 m

Answer Daily Practice Paper - 3


1. (2) 2. (4) 3. (2) 4. (2) 5. (2)
6. (3) 7. (3) 8. (3) 9. (1) 10. (1)
11. (4) 12. (2) 13. (3)

Note

Corporate Office : B-58, Goal Building, Budha Colony, Patna-1, Help line : 9334594165/66/67 Website : www.goalinstitute.org
CHAPTER-05

MOTION IN TWO DIMENSIONS


5. MOTION IN TWO DIMENSIONS :
If a body moves in a plane defined by any two axis of three, the motion is called two dimensional
motion. Projectile Motion and circular motion of a body are the examples of two dimensional motion.
5.1 PROJECTILE MOTION :
Any object that is given an initial velocity obliquely and that subsequently follows a path determined
by the gravitational force acting on it, is called projectile and motion is called projectile motion.
It is an example of two dimensional motion (y and x axis), follows a parabolic path.

ax = 0 u y = usin
u
usin H = + ay= –g
 ucos
0 ucos Horizontal motion Vertical motion
R

5.2 PROJECTILE THROWN AT AN ANGLE WITH HORIZONTAL :


Consider a projectile thrown with a velocity u making an angle with the horizontal. If we consider
projectile motion as the combination of horizontal and vertical motion. Therefore horizontal direction
vertical direction
Horizontal Direction Vertical Direction
(1) ux = ucos  (constant) (1) uy = usin
(2) ax = 0 (2) ay = –g

u
usin

H

ucos
R

(1) Time of Flight : For vertical displacement = 0, we have


1
0  usinT  gT 2
2

T  2usin
g
Kinematics [ 40 ]
(2) Relation between velocity of projectile and angle at any instant :
As we know, the horizontal component of velocity remains unchanged, so we have
v x = ux
v
vcos  = u cos u 
ucos  
v
cos 
(3) Horizontal Range :
R = ux·T
2usin
R  ucos ·
g

u2 sin2
R
g
Ex. Find the angle of projection for which range is maximum.
n
Sol . For range, we have
u2 sin2 
R .
g
for sin 2= 1, R is maximum
We get
 = 45° (angle for maximum range)
and
2
Rmax  u
g
Ex. Prove that for given value of u range at angle  and range at angle 90 – are equal although time
of flight and height may be different.
n
Sol . For angle Range is
2u2 sin.cos  u2 sin2 
R1  
g g
For angle 90 – 
u =90 – 
Range is
u H
2u2 sin(90 )cos(90 ) u2 sin2 
R2  
g g 

and R1  R 2
(4) Maximum height : Using equation v2 = u2 + 2as
We have,
0 = u2sin2– 2gH.
2 2
H  u sin 
2g

Corporate Office : B-58, Goal Building, Budha Colony, Patna-1, Help line : 9334594165/66/67 Website : www.goalinstitute.org
Kinematics [ 41 ]
5.3 PROJECTILE THROWN PARALLEL TO HORIZONTAL :
Consider a projectile thrown horizontally from height h with velocity.

2h
(1) Time of flight t 
g u

2h
(2) Horizontal Range R  u g h
ux
uy v
(3) Velocity at vertical depth h :
R
vx  u

1
(4) Displacement s  x2  y2 where x  ux t and y  0  gt 2
2
(5) Velocity at time t
v  v 2x  v 2y where v x = u and vy = 0 + gt

vy
v  u2  g2t 2 and tan 
vx
Ex. An aeroplane is flying at a constant horizontal velocity of 600 km/hr at an elevation of 6 km towards
a point directly above the target on the earth’s surface.
At an appropriate time, the pilot releases a ball so that it strikes the target at the earth. The ball will
appear to be falling

(1) On a parabolic path as seen by pilot in the plane


(2) Vertically along a straight path as seen by an observer on the ground near the target
(3) On a parabolic path as seen by an observer on the ground near the target
(4) On a zig-zag path as seen by pilot in the plane
Soln. (3) The path of the ball appears parabolic to a observer near the target because it is at rest. But
to a Pilot the path appears straight line because the horizontal velocity of aeroplane and the
ball are equal, so the relative horizontal displacement is zero.
Ex. A ball rolls off top of a staircase with a horizontal velocity u m/s. If the steps are h metre high and
b mere wide, the ball will just hit the edge of nth step if n equals to

hu2 u2h 2hu2 2u2g


(1) gb2 (2) gb2 (3) gb2 (4)
hb2
Corporate Office : B-58, Goal Building, Budha Colony, Patna-1, Help line : 9334594165/66/67 Website : www.goalinstitute.org
Kinematics [ 42 ]

gx 2
Soln. (3) By using equation of trajectory y  for given condition
2u2

g(nb)2 u
nh 
2u2 h
b
2hu2 nh
 n
gb 2
nb

IMPORTANT POINT–1
(a) In projectile motion vertical component of velocity changes but horizontal component of velocity
remains always constant and equal to u cos.
(b) For complementary angles of projection and 90 – range is same.

2h
(c) Time to reach the ground for the projectile is t  g when projected horizontally from height h.

T1
(d) For complementary angles of projection and 90 – ratio of time of flight T  tan
2

(e) Product of time of flight for complementary angle of projection is


2R
T1·T2  .
g

4H
(f) Relation between range and Maximum height tan , for H R  tan 4.
R
If a person can throw a projectile to maximum distance R. The maximum height to which it will
R
rise is H .
4
(g) v cos = u cos where u and are the initial value and v and are the value at any instant.

 u  gt 
(h) Angle between the velocity and the x-axis is given by   tan–1 y 
 ux 

(i) Change in velocity between point of projection and highest point, U = –usin j

(j) Change in velocity with projectile returns to ground is –2usin j
(k) If projected horizontally or released from horizontally moving aeroplane has zero initial velocity
in vertical direction

Corporate Office : B-58, Goal Building, Budha Colony, Patna-1, Help line : 9334594165/66/67 Website : www.goalinstitute.org
Kinematics [ 43 ]

Solved Example – 1
Ex.1. For a given velocity, a projectile has the same range R for two angles of projection if t1 and t2 are
the times of flight in the two cases then

1 1
(1) t 1t 2  R2 (2) t 1t 2  R (3) t 1t 2  (4) t 1t 2 
R R2

2R
Soln. (2) As we know for complementary angles t1t 2 
g

 t1t2  R.
Ex.2. A body is thrown with a velocity of 9.8 m/s making an angle of 30o with the horizontal. It will hit
the ground after a time
(1) 1.5 s (2) 1s (3) 3 s (4) 2 s

2u sin 2 9.8  sin 30


Soln. (2) T
g

9.8

 1sec
Ex.3. A particle is projected from a point O with a velocity u in a direction making an angle  upward
with the horizontal. After some time at point P it is moving at right angle with its initial direction of
projection. The time of flight from O to P is

usin  ucosec  utan usec 


(1) g (2) g (3) g (4) g


Soln. (2) When body projected with initial velocity u by making angle  with the horizontal. Then

after time t, (at point P) it’s direction is perpendicular to u .

90°
u P (90 – )
u sin

 v cos v
O u cos

Magnitude of velocity at point P is given by v = u cot.


For vertical motion : Initial velocity (at point O) uy = u sin
Final velocity (at point P)

v y  vcos 

 v y  u cot cos 
Time of flight (from point O to P) = t

Corporate Office : B-58, Goal Building, Budha Colony, Patna-1, Help line : 9334594165/66/67 Website : www.goalinstitute.org
Kinematics [ 44 ]
Applying first equation of motion vy = uy – gt
–u cotcos= u sin– gt

u sin  u cot  cos 


 t
g

u u cosec
 sin2   cos2  
g sin   g
Ex.4. A ball is projected upwards from the top of tower with a velocity 50 ms–1 making angle 30o with
the horizontal. The height of the tower is 70 m. After how many seconds from the instant of
throwing will the ball reach the ground
(1) 2.33 sec (2) 5.33 sec (3) 6.33 sec (4) 9.33 sec
Soln. (3) Formula for calculation of time to reach the body on the ground from the tower of height ‘h’
2gh
(If it is thrown vertically up with velocity u) is given by t  u 1 2 
g u 

u sin30o u
30o

70 m

So we can resolve the given velocity in vertical direction and can apply the above formula.
Initial vertical component of velocity u sin  = 50 sin 30 = 25 m/s.

25  2×9.8 70 
 t 1 1
9.8  (25)2 
= 6.33 sec.
Ex.5. A particle is thrown with velocity u at an angle  from the horizontal. Another particle is thrown
with the same velocity at an angle  from the vertical. The ratio of times of flight of two particles
will be
(1) tan 2  : 1 (2) cot 2  : 1 (3) tan  : 1 (4) cot  : 1

2u sin
Soln. (3) For first particles angle of projection from the horizontal is . So T1 
g
For second particle angle of projection from the vertical is . It mean from the horizontal is
(90   ).

2u sin(90 ) 2u cos 


 T2   .
g g

So ratio of time of flight T1  tan .


T2

Corporate Office : B-58, Goal Building, Budha Colony, Patna-1, Help line : 9334594165/66/67 Website : www.goalinstitute.org
Kinematics [ 45 ]
Ex.6. The friction of the air causes vertical retardation equal to one tenth of the acceleration due to
gravity (Take g = 10 ms–2). The time of flight will be decreased by
(1) 0% (2) 1% (3) 9% (4) 11%
2u sin
Soln. (3) T
g

g
g
 T1 g2 10 11
  
T2 g1 g 10

T1  T2 1
Fractional decrease in time of flight  
T1 11
Percentage decrease = 9%
Ex.7. Which of the following sets of factors will affect the horizontal distance covered by an athlete in
a long–jump event
(1) Speed before he jumps and his weight
(2) The direction in which he leaps and the initial speed
(3) The force with which he pushes the ground and his speed
(4) The direction in which he leaps and the weight

(Velocity of projection)2 sin2 (Angle of projection)


Soln. (2) Because Range 
g
Ex.8. A cricketer can throw a ball to a maximum horizontal distance of 100 m. The speed with which
he throws the ball is (to the nearest integer)
(1) 30 ms –1 (2) 42 ms –1 (3) 32 ms –1 (4) 35 ms –1

Soln. (3) u2
Rmax  100 (when  = 45°)
g

 u  1000  31.62 m/s


Ex.9. If two bodies are projected at 30o and 60o respectively, with the same velocity, then
(1) Their ranges are same (2) Their heights are same
(3) Their times of flight are same (4) All of these
Soln. (1) Because these are complementary angles.
Ex.10. Four bodies P, Q, R and S are projected with equal velocities having angles of projection 15o,
30o, 45o and 60o with the horizontal respectively. The body having shortest range is
(1) P (2) Q (3) R (4) S
Soln. (1) Range of projectile will be minimum for that angle which is farthest from 45°.
Ex.11. A particle covers 50 m distance when projected with an initial speed. On the same surface it will
cover a distance, when projected with double the initial speed
(1) 100 m (2) 150 m (3) 200 m (4) 250 m

Corporate Office : B-58, Goal Building, Budha Colony, Patna-1, Help line : 9334594165/66/67 Website : www.goalinstitute.org
Kinematics [ 46 ]

u2 sin 2
Soln. (3) R
g

2 2
R2  u2  2u
 R  u2 so R   u   u
1  1
 
 R2 = 4R1
= 4 × 50 = 200m
Ex.12. A projectile thrown with a speed v at an angle  has a range R on the surface of earth. For same
v and , its range on the surface of moon will be
(1) R/6 (2) 6R (3) R/36 (4) 36 R

u2 sin 2
Soln. (2) R
g  R  1/g

Rmotion gEarth g 1 


 6
REarth gMotion  moon  6 gEarth 

 RMoon = 6REarth = 6R
1
Ex.13. The speed of a projectile at the highest point becomes times its initial speed. The horizontal
2
range of the projectile will be

u2 u2 u2 u2
(1) g (2) 2g (3) 3g (4) 4g

Soln. (1) Velocity at the highest point is given by ucos   u (given)


2
  = 45o

2 u2 sin(2 45) u2
Horizontal range R  u sin2  
g g g
Ex.14. A large number of bullets are fired in all directions with same speed u. What is the maximum
area on the ground on which these bullets will spread

u2 u4 2u
2
2u
2
(1)  (2)  (3)  g2 (4)  g2
g g2
Soln. (2) The maximum area will be equal to area of the circle with radius equal to the maximum
range of projectile
2
2 u2  2 u4
Maximum area r  (Rmax )   g    2
  g

[As r  R max  u2 /g for   45]

Corporate Office : B-58, Goal Building, Budha Colony, Patna-1, Help line : 9334594165/66/67 Website : www.goalinstitute.org
Kinematics [ 47 ]
Ex.15. A projectile thrown with an initial speed u and angle of projection 15o
to the horizontal has a
range R. If the same projectile is thrown at an angle of 45o to the horizontal with speed 2u, its
range will be
(1) 12 R (2) 3R (3) 8 R (4) 4 R

Soln. (3) u2 sin2  R  u2 sin2


R
g

2
R2  u2   sin22 

R1  u1   sin2 
 1

2
   sin
R1 2u
u sin30
90  8R
1

Ex.16. A particle is projected with a velocity v such that its range on the horizontal plane is twice the
greatest height attained by it. The range of the projectile is (where g is acceleration due to
gravity)

4v 2 4g v2 4v 2
(1) 5g (2) (3) g (4) 5g
5v 2
Soln. (1) We know R = 4H cos
2H = 4H cot
1 2 1
 cot  ; sin ; cos  [As R = 2H given]
2 5 5

v 2 ·2sin·cos 
Range 
g

2v 2 2 · 1
 5 5  4v 2
g 5g
Ex.17. The range R of projectile is same when its maximum heights are h1 and h2. What is the relation
between R and h1 and h2
(1) R  h1h2 (2) R  2h1h2 (3) R  2 h1h2 (4) R  4 h1h2

Soln. (4) For equal ranges body should be projected with angle  or (90 o   ) from the horizontal.

u2 sin2  u2 cos2 
And for these angles : h1  andh2 
2g 2g

4 2 2 2 2
by multiplication of both height : h1h2  u sin  2cos   1  u sin2 
4g 16 g  
 16h1h2 = R2
 R  4 h1h2

Corporate Office : B-58, Goal Building, Budha Colony, Patna-1, Help line : 9334594165/66/67 Website : www.goalinstitute.org
Kinematics [ 48 ]
Ex.18. A grasshopper can jump maximum distance 1.6 m. It spends negligible time on the ground. How
far can it go in 10 seconds
(1) 5 2m (2) 10 2 m (3) 20 2 m (4) 40 2 m
Soln. (3) Horizontal distance travelled by grasshopper will be
maximum for  = 45°

u2
Rmax  1.6m
g
 u  4m/s. 1.6m

Horizontal component of velocity of grasshopper u cos  = 4 cos 45 = 2 2 m/s


Total distance covered by it in10 sec.
S  u cos   t

 2 2  10  20 2 m
Ex.19. For a projectile, the ratio of maximum height reached to the square of flight time is (g = 10 ms–2)
(1) 5: 4 (2) 5: 2 (3) 5 : 1 (4) 10 : 1
u2 sin2  2u sin
Soln. (1) H
2g
and T 
g

H u2 sin2  /2g
 
T 2 4u2 sin2  /g2

g 10 5
  
8 8 4
Note

Corporate Office : B-58, Goal Building, Budha Colony, Patna-1, Help line : 9334594165/66/67 Website : www.goalinstitute.org
Kinematics [ 49 ]

Daily Practice Paper – 1


1. In a projectile motion, the velocity
(1) Is always perpendicular to the acceleration.
(2) Is never perpendicular to the acceleration.
(3) Is perpendicular to the acceleration for one instant only.
(4) Is perpendicular to the acceleration for two instant.
2. A cricket ball is thrown with a velocity of 15 m/s at an angle of 30o with the horizontal. The time of
flight of the ball will be (g = 10 m/s2)
(1) 1.5 s (2) 2.5 s (3) 3.5 s (4) 4.5 s
3. The range of a particle when launched at an angle of 15o with the horizontal is 1.5 km. What is the
range of the projectile when launched at an angle of 45o to the horizontal
(1) 1.5 km (2) 3.0 km (3) 6.0 km (4) 0.75 km
4. Which of the following does not affect the maximum height attained by the projectile
(1) Magnitude of initial velocity (2) Acceleration of the projectile
(3) Angle of projection (4) Mass of the projectile
5. A person can throw a stone to a maximum distance of 100 m, the greatest height to which he can
throw the stone is
(1) 100 m (2) 75 m (3) 50 m (4) 25 m
6. A stone is thrown at an angle  to the horizontal reaches a maximum height h. The time of flight of
the stone is

(1) (2h sin )/g (2) 2 (2h sin )/g (3) 2 (2h)/g (4) (2h)/g

7. It was calculated that a shell when fired from a gun with a certain velocity and at an angle of
5
elevation of radians should strike a given target. In actual practice it was found that a hill just
36
intervened in the trajectory. At what angle of elevation should the gun be fired to hit the target

5 11 7 13 
(1) rad (2) rad (3) rad (4) rad
36 36 36 36
8. A projectile is thrown in the upward direction making an angle of 60o with the horizontal direction
with a velocity of 147 ms–1. Then the time after which its inclination with the horizontal is 45o is
(1) 15 s (2) 10.98 s (3) 5.49 s (4) 2.745 s
9. An aeroplane is flying horizontally with a velocity of 600 km/h and at a height of 1960m. When it is
vertically at a point A on the ground, a bomb is released from it. The bomb strikes the ground at
point B. The distance AB is
(1) 1200 m (2) 0.33 m (3) 3.33 km (4) 33 km
10. Two bullets are fired with horizontal velocities of 50 m/s and 100 m/s from two guns at a height of
19.6 m. Which bullet will strike first
(1) First (2) Second (3) Simultaneously (4) None of these
Corporate Office : B-58, Goal Building, Budha Colony, Patna-1, Help line : 9334594165/66/67 Website : www.goalinstitute.org
Kinematics [ 50 ]
11. Two paper screens (1) and (2) are separated by a distance of 100 m. A bullet pierces (1) and (2)
the hole (2) is 10 cm below the hole (1). If the bullet is travelling horizontally at the time of hitting
(1). Then velocity of the bullet at (1) is
(1) 100 m/sec (2) 200 m/sec (3) 600 m/sec (4) 700 m/sec
12. From the top of a tower 19.6 m high, a ball is thrown horizontally. If the line joining the point of
projection to the point where it hits the ground makes an angle of 45o with the horizontal, then the
initial velocity of the ball is
(1) 9.8 ms–1 (2) 4.9 ms–1 (3) 14.7 ms–1 (4) 2.8 ms–1
13. A boy throws a ball with a velocity V0 at an angle  to the horizontal. At the same instant he starts
running with uniform velocity to catch the ball before it hits the ground. To achieve this, he should
run with a velocity of

(1) V0 cos  (2) V0 sin  (3) V0 tan  (4) v 20 tan


14. A ball is projected from the ground at a speed of 10 m/s making angle 30o with the horizontal.
Another ball is simultaneously released from a point on the vertical line along the maximum height
of the projectile. Both the balls collide at the maximum height of the projectile. What was the initial
height of the second ball
(1) 1.0 m (2) 1.25 m (3) 2.0 m (4) 2.5 m

Answer Daily Practice Paper - 1


1. (3) 2. (1) 3. (2) 4. (4) 5. (3)
6. (3) 7. (4) 8. (3) 9. (3) 10. (3)
11. (4) 12. (1) 13. (1) 14. (4)

Note

Corporate Office : B-58, Goal Building, Budha Colony, Patna-1, Help line : 9334594165/66/67 Website : www.goalinstitute.org
Kinematics [ 51 ]

5.4 EQUATION OF TRAJECTORY :


The path followed by a particle during its motion is called trajectory. For projectile motion we have
1
x  ux t and y  uy t  gt 2
2
x = u cos t …(i)

1
y  u sint  gt 2 …(ii)
2
Substituting the value of t in equation (2)
2
y  usin· x – 1g x
ucos  2 ucos  
gx 2
y  x tan –
2u cos 2 
2

Comparing with y = ax – bx2, it is an equation of parabola. Where ‘a’ and ‘b’ are the constant.
This is an equation of trajectory of projectile (Parabolic path)
5.5 ENERGY OF PROJECTILE :
When a projectile moves upward its kinetic energy decreases, potential energy increases but the
total energy always remain constant.
If a body is projected with initial kinetic energy (K = 1/2 mu2), with angle of projection  with the
horizontal then at the highest point of trajectory
(1) Kinetic energy
1
 m (u cos )2
2
1
 mu2 cos 2 
2
 K'  Kcos 2 

(2) Potential energy

u2 sin2  1  As H  u2 sin2 
 mgH  mg  mu2 sin2  
2g 2  2g 
(3) Total energy
= Kinetic energy + Potential energy

1 1
 mu2 cos2   mu2 sin2 
2 2

1
 mu2 = Energy at the point of projection.
2
This is in accordance with the law of conservation of energy.

Corporate Office : B-58, Goal Building, Budha Colony, Patna-1, Help line : 9334594165/66/67 Website : www.goalinstitute.org
Kinematics [ 52 ]
5.6 CHANGE IN MOMENTUM :
(1) Change in momentum between projection point and highest point
   
 P  Pf  Pi  – mu sin j
(2) Change in momentum between initial point and final point for the complete projectile motion
   
 P  Pf  Pi  –2musin j
IMPORTANT POINTS–2
gx 2
(a) The equation of trajectory y  x tan  
2u2 cos 2 

Taking tancommon and doing some manipulation we can write y = x tan 1  Rx 


(b) The body is projected horizontally, equation of trajectory
1 x2
y g
2 u2
  
(c) If a body is projected with velocity u  ux i  uy j , then

u
–1 y
u  u2x  u2y and angle  tan u then use basic formula
x

(d) The angle of elevation  is given by


y
 tan–1 .
x

For maximum height tan 1 tan


2

Solved Example–2
BASED ON TRAJECTORY AND VECTOR FORM
Ex.20. The trajectory of a projectile is represented by y  3x  gx 2 /2. . The angle of projection is
(1) 30o (2) 45o (3) 60o (4) None of these
Soln. (3) By comparing the coefficient of x in given equation with standard equation
gx 2
y  xtan tan 3   60
2u2 cos 2 

Ex.21. The path followed by a body projected along y-axis is given as by y = 3 x – (1/2)x 2, if
g = 10 m/s, then the initial velocity of projectile will be – (x and y are in m)
(1) 3 10 m/ s (2) 2 10 m/ s (3) 10 3 m/ s (4) 10 2 m/ s

Corporate Office : B-58, Goal Building, Budha Colony, Patna-1, Help line : 9334594165/66/67 Website : www.goalinstitute.org
Kinematics [ 53 ]
Soln. (2) By comparing the coefficient of x2 in given equation with standard equation
gx 2
y  x tan   .
2u cos2 
2

g 1

2u2 cos 2  2
Substituting  = 60°
we get u  2 10 m/ sec.
 
Ex.23. A projectile is projected with velocity 3 i  4 j m/s from ground find the range of projectile.

Soln. V  32  4 2  5 m/s

4
 tan–1
3
  53
u2 sinx 52 sin106 
for range R  g

10
 2.4m
Ex.24. A particle is projected at an angle of 30° with speed 20 m/s
(i) Find out position vector of the particle after 1s
(ii) Find out angle between velocity vector and position vector at t = 1s
3
Soln. (i) s x = u cos  t  20 × × t  10 3m
2
1 1
s y  u sin  t  × 10 × t 2  20 × × 1  5(1)2  5m
2 2
 
Position vector  10 3 i  5 j

(ii) v x 10 3 i
v y  uy  ay t  10 10  0
   
V  10 3 i v · s  v s cos 

10 3×10 3
cos 
10 3 × 325

10 3 3
 2
325 13

 3
 cos –1 2 
 13 

Corporate Office : B-58, Goal Building, Budha Colony, Patna-1, Help line : 9334594165/66/67 Website : www.goalinstitute.org
Kinematics [ 54 ]

Daily Practice Paper – 2


1. At the top of the trajectory of a projectile, the directions of its velocity and acceleration are
(1) Perpendicular to each other (2) Parallel to each other
(3) Inclined to each other at an angle of 45o (4) Antiparallel to each other
2. A man projects a coin upwards from the gate of a uniformly moving train. The path of coin for the
man will be
(1) Parabolic (2) Inclined straight line
(3) Vertical straight line (4) Horizontal straight line
3. A particle moves in the X-Y plane according to the law x = kt and y =kt (1 – t), where k and  are
positive constants and t is time. What is the equation of trajectory of the particle
x2 x 2
(1) y = kx (2) y x (3) y  (4) y = x
k k
4. An object is projected with a velocity of 20 m/s making an angle of 45o with horizontal. The equation
for the trajectory is h = Ax – Bx2 where h is height, x is horizontal distance, A and B are constants.
The ratio A : B is (g = 10 m/s2)
(1) 1 : 5 (2) 5 : 1 (3) 1 : 40 (4) 40 : 1
5. o
A particle of mass m is thrown at an angle of 45 with the horizontal with a momentum p. The
magnitude of its momentum at the top of its parabolic path will be
(1) p (2) 2p (3) p/ 2 (4) p/2
6. A ball of mass m and kinetic energy E is thrown at an angle of 45o with the horizontal. The momentum
at the highest point of its path will be
(1) mE (2) mE/2 (3) 2mE (4) Zero
7. A cricket ball is hit at 30o with the horizontal with kinetic energy K. The kinetic energy at the highest
point is
(1) Zero (2) K/4 (3) K/2 (4) 3K/4
8. The ratio of kinetic energies of two balls at the highest point, one of them is projected vertical
upward and other with some angle
(1) 4 : 3 (2) 3 : 4 (3)  (4) 0
9. A body is projected such that the K.E. at the topmost position is half of the initial K.E. What is its
angle of projection with the horizontal
(1) 30o (2) 60o (3) 75o (4) 45o
10. A body is thrown with initial kinetic energy 100J at an angle o to the horizontal. If its kinetic energy
at the top is 25J, then the angle of projection is
(1) 30o (2) 45o (3) 60o (4) 90o

Answer Daily Practice Paper – 2


1. (1) 2. (3) 3. (2) 4. (4) 5. (3)
6. (1) 7. (4) 8. (4) 9. (4) 10. (3)

Corporate Office : B-58, Goal Building, Budha Colony, Patna-1, Help line : 9334594165/66/67 Website : www.goalinstitute.org
Kinematics [ 55 ]

5.7 PROJECTION FROM A MOVING BODY :


Consider a boy standing on a trolley who throws a ball with speed u at an angle with the horizontal.
Trolley moves horizontally with constant speed v.
Case (i) : When ball is projected in the direction of motion of the trolley, horizontal component
of ball’s velocity = u cos+ v, initial vertical component of ball’s velocity = u sin
Case (ii) : The ball is projected opposite to the direction of motion of the trolley Horizontal
component of ball’s velocity = u cos– v
Initial vertical component of ball’s velocity = u sin
Case (iii) : The ball projected upwards from a platform moving with speed v upwards.
Horizontal component of ball’s velocity = u cos
Initial vertical component of ball’s velocity = u sin+ v
Case (iv) : The ball projected upwards from a platform moving with speed v downwards.
Horizontal component of ball’s velocity = u cos
Initial vertical component of ball’s velocity = u sin– v
Ex. A boy standing on a long railroad car throws a ball straight upwards. The car is moving on the
horizontal road with an acceleration of 1 m/s2 and the projection speed in the vertical direction is
9.8 m/s. How behind the boy will the ball fall on the car ?
2u
Soln. Let the initial velocity of car be ‘u’, t   2 sec
g
where u = component of velocity in vertical direction
1 2
xc  u  2   2  2u  2
2
where x c = distance travelled by car; x b = distance travelled by ball
x b = 2u; x c – x b = 2u + 2 – 2u = 2m
Ex. A person is standing on a truck moving with a constant velocity of 14.7 m/s on a horizontal road.
The man throws a ball in such a way that it returns to the truck after the truck has moved 58.8m.
Find the speed and the angle of projection (1) as seen from the truck, (2) as seen from the road.
n
Sol . (1) 19.6 m/s upward (2) 24.5 m/s at 53° with horizontal)
5.8 RELATIVE MOTION BETWEEN TWO PROJECTILES :
Let us now discuss the relative motion between two projectiles or the path observed by one projectile
of the other. Suppose that two particles are projected from the ground with speed u1 and u2 at
angles 1 and 2 as shown in figure.

Y Y

u1 u2

1 2
X X

Corporate Office : B-58, Goal Building, Budha Colony, Patna-1, Help line : 9334594165/66/67 Website : www.goalinstitute.org
Kinematics [ 56 ]
Relative acceleration between them is zero because
ar = g – g
ar = 0
relative velocity along x-axis
ux = u1cos 1 – u2cos 2 uy
relative velocity along y-axis
uy = u1sin1 – u2sin2

Hence, relative motion of 1 with respect to 2 is a
ux
straight line at an angle

 uy 
 tan–1 
 ux 
5.9 CONDITION OF COLLISION OF TWO PROJECTILES :
uB
Q
uA 2
Q (uAB )y uAB
h2–h1
P 1
h2 
P
h1 h1 (uAB) x h2
S

Let the two particles be projected simultaneously from two different heights h1 and h2 with speed
uA and uB at angles 1 and 2 respectively as shown in figure. Then, the two particles may collide
 
in mid air if the relative velocity of two particles uAB or uBA is along the straight line PQ,

The time of collision of the two particles is given by,


length of straight line
t  
uAB or uBA

At the time of collision, co-ordinates of particles should be identical i.e. xA = x B and y A = y B


Ex. A particle A is projected with an initial velocity of 60m/s at an angle 30° to the horizontal. At the
same time a second particle B is projected in opposite direction with initial speed of 50m/s from a
point at a distance of 100m from A. If the particles collide in air, find (i) the angle of projection  of
particle B(ii) time when the collision takes place and (iii) the distance of P from A, where collision
occurs, (g = 10 m/s2)

50 m/s
60 m/s
o
30 
A B
x

Soln. (1) Taking x and y-directions as shown in figure.


 
Here, a A  g j

Corporate Office : B-58, Goal Building, Budha Colony, Patna-1, Help line : 9334594165/66/67 Website : www.goalinstitute.org
Kinematics [ 57 ]

 
a B g j

uAx  60cos 30  30 3m/ s


uAy = 60 sin 30° = 3 m/s
uBx = 5 cos 
and uBy = 50 sin 
 
Relative acceleration between the two is zero as a a A  aB .Hence, the relative motion between

the two is uniform. It can be assumed that B is at rest and A is moving with VAB . Hence, the two

particle will collide if VAB is along AB. This is possible only when
y
uAy = uBy
i.c, component of relative velocity along y-axis should be zero.
or, 30 = 50 sin
 = sin–1 (3/5)
x

(2) Now, uAB  uAx  uBx uAB

 (30 3  50 cos  )m / s
 4
  30 3  50 ×  m / s
 5
 30 3  40m/s
Therefore, time of collision is
AB 100
t  
uAB 30 3  40
or, t = 1.09 s
(3) Distance of point P from A where collision takes place is
2


s  (uAx t)2  u Ay t  1gt 2
2 
2
 1 
 (30 3 × 1.09)2   30  1.09  × 10 × 1.09 × 1.09 
 2 
or, s = 62.54m
Ex. Two particles are separated at a horizontal distance x as shown in figure. They are projected at
the same time as shown in figure with different initial speed. The time after which the horizontal
distance between the particles become zero is
u
u/ 3
30° 60°
A B
x
(1) u/2x (2) x/u (3) 2y/x (4) u/x
Corporate Office : B-58, Goal Building, Budha Colony, Patna-1, Help line : 9334594165/66/67 Website : www.goalinstitute.org
Kinematics [ 58 ]
Soln. (2) Let x 1 and x 2 are the horizontal distances travelled by particle A and B respectively in time t.
u …(i) and x2  ucos60 × t …(ii)
x1  ·cos30 ×t
3

u
x1  x2  ·cos30 t  ucos60 t  ut
3

 x  ut t  x /u
IMPORTANT POINTS–3
(1) Relative acceleration is zero between two projectiles.
(2) Motion of a projectile with respect to other projectile is uniform motion.
(3) For collision separation becomes zero in x and y axis.
(4) Hence to collide the projectiles x = uxr.t

Note

Corporate Office : B-58, Goal Building, Budha Colony, Patna-1, Help line : 9334594165/66/67 Website : www.goalinstitute.org
Kinematics [ 59 ]
5.10 R E L A TIV E V E L OC ITY :
Imagine yourself standing by the side of a road observing two car A and B both moving towards
right with speed 10m/s. Let us consider car A for a moment. According to you its velocity is
+10m/s. But an observer in car B will measure the speed of car A as 0m/s. His observation about
the velocities are different. According to him car A is at rest and road itself is moving with –10m/s
(i.e. backward)

(A) (B)
10m/s 10m/s

Hence whenever we speak about the velocity of any moving object, we must also specify the
reference from whose we are observing it.
Velocities observed from the ground will be called as velocities relative to the ground. Velocities
observed from the car B will be called as velocities relative to the car B.

Hence, velocity of car A relative to the ground VAG 10m/ s


Velocity of car A relative to the car B VAB  0 m/s
  
In general we have the relation VAB  VAG  VBG
Ex. A man is running down the side of a road with speed of 6m/s. A cyclist is going in the same
direction with speed of 8m/s. Find velocity of running man with respect to cyclist.

Soln. Velocity of running man v m

Velocity of cyclist = v c
Velocity of running man with respect to cyclist
    
v mc  v m  v c  6 i  8 i

v mc  2 i
i.e. to the cyclist it will appear that the man is running with speed 2m/s in backward direction.
Ex. A particle A is moving towards + x – axis with speed 7 m/s. Another particle B is going
towards – x axis with speed 3m/s. Find the velocity B relative to A.
  
Soln. vBA  v B  v A
  
 3 i m/s  7 i m/s  10 i m/s

Corporate Office : B-58, Goal Building, Budha Colony, Patna-1, Help line : 9334594165/66/67 Website : www.goalinstitute.org
Kinematics [ 60 ]

Daily Practice Paper - 3


1. A car travelling at 60km/h overtakes another car travelling at 42 km/h. Assuming each car to be
5.0 m long, find the time taken during the overtake and the total road distance used for the overtake.
2. Two railway tracks are parallel to west-east direction. A long one track, train A moves with a speed of
54km/h from west to east while along the second track train B moves with a speed of 90km/h from east
to west. Find (i) the relative velocity of B w.r.t. A and (ii) relative velocity of ground w.r.t. B.
3. A police van moving on a highway with a speed of 30km/h fires bullet at a thief’s car speeding away
in same direction with a speed of 192 km/h. If the muzzle speed of the bullet is 150 m/s with what
speed does the bullet hit the thief’s car.
4. An object A is moving with 10m/s and B is moving with 5 m/s in the same direction of positive x-
axis A is 100 m behind B as shown. Find time taken by A to meet B.

10m/s 5m/s
A B

100 m distance

5. An object A is moving with 5m/s and B is moving with 10m/s in the same direction of positive x-axis
simultaneously from the same point as shown. Find the separation between them after 10 sec.

A 5m/s B 10m/s

6. On a two-lane road, car A is travelling with a speed of 36 km /h. Two cars B andC approach car
A in opposite directions with a speed of 54 km /h each. At a certain instant, when the distance AB
is equal to AC, both being 1 km, B decides to overtake A before C does. What minimum acceleration
of car B is required to avoid an accident ?
7. Two towns A and B are connected by a regular bus service with a bus leaving in either direction
every T minutes. A man cycling with a speed of 20 km /h in the direction A to B notices that a bus
goes past him every 18 min in the direction of his motion, and every 6 min in the opposite
direction. What is the period T of the bus service and with what speed (assumed constant) do the
buses ply on the road?
8. Two cars A and B are moving with constant velocity 10m/s and 5m/s respectively. At a particular
moment separation between cars is 150m. Find the time after which car A will overtake car B.
9. A ball is thrown downward with a speed of 20m/s from the top of a building 150 m high and
simultaneously another ball is thrown vertically upward with a speed of 30m/s from the foot of the
building. Find the time when the balls will meet. (g = 10m/s2)
10. Two cars C1 and C2 moving in the same direction on a straight road with velocities 12m/s and
10 m/s respectively. When the separation between the two was 200m, C2 started accelerating to
avoid collision. What is the minimum acceleration of car C2 so that they do not collide.

Corporate Office : B-58, Goal Building, Budha Colony, Patna-1, Help line : 9334594165/66/67 Website : www.goalinstitute.org
Kinematics [ 61 ]
11. Find the time after which A overtakes B.
vA= 10m/s vB= 5m/s

A B

a = 8m/s2 aB= 2m/s2


100 m
12. A stone is dropped from the top of the building and at the same time a second stone is thrown
vertically upward with a speed of 20m/s. If height of building is 60m. Find the time after which the
stones collide. Stone are moving in the same line (g = 10 m/s2).

Answer Daily Practice Paper-3


1. 2 sec 115/3 m 2. 40 m/s, 25 m/s 3. 105 m/s
4. 20 sec 5. 50 m 6. 1 m/s 2
7. 9 min 8. 30 sec 9. 3 sec
10. 0.01 m/s 2 11. 5 sec 12. 3 sec

Note

Corporate Office : B-58, Goal Building, Budha Colony, Patna-1, Help line : 9334594165/66/67 Website : www.goalinstitute.org
Kinematics [ 62 ]
5.11 RELATIVE VELOCITY WHEN PARTICLES MOVE IN DIFFERENT LINES :
Mathematically we have
   
v AB  v A  v B –vB

v AB  v 2A  vB2  2v A v B cos  
   vA

direction of A as seen by B as shown in figure  v AB . vAB = vA –vB

Ex. A balloon is moving horizontally in air with speed of 


vB
5m/s towards north. A car is moving with 5 m/s towards
east. If a person sitting inside the car sees the balloon,
what will be observe

north

v 
v
b 5 2 b

Soln. 
v 5m/s
C
west

–vc

      
We have, v b  5 j and v c  5 i ,v bc  v b  v c

To calculate the balloons’s velocity as seen from the car, we have to view the situation by placing
ourselves inside the car. For calculation relative to car, we will impose the reversed velocity of car
(–v c) on the balloon velocity (vb). The net effect of these gives the velocity of the balloon as seen
from the car.

Hence the velocity of balloon as seen from the car is 5 2 m/s towards NW.
Ex. A man walking eastward at 6km/hr finds that the wind seems to blow directly from north. On
doubling his velocity, the wind appears to come N 30°E. Find the velocity of the wind.
Soln. Actual velocity of the man = 6 km/hr eastward.
N N
12km/hr 30°
O 6km/hr A
W C E

S S
°E
30
nN
ti o
ec
D ir

B
Corporate Office : B-58, Goal Building, Budha Colony, Patna-1, Help line : 9334594165/66/67 Website : www.goalinstitute.org
Kinematics [ 63 ]
The direction of the relative velocity of the wind in this case is North to south.
 
If OA represents the velocity of man and AB represents the relative velocity of the wind, w.r.t. the

man then velocity of man + relative velocity of wind w.r.t. the man = velocity of wind = OB (say).
It is also given that when the velocity of the man is doubled (i.e. 12 km/hr) the wind seems to blow

from a direction N30°E. Representing this by vector OC New velocity of man = 12km/hr.
The direction of the relative velocity of the wind w.r.t. the man in the
case is CB.

The two directions of the relative velocity meet at B. Hence OB should give the real velocity of the
wind. From the geometry of the Figure, it is clear that OBC is an equilateral triangle. Hence the
magnitude of the real velocity of the wind = 12km/hr. Its direction is towards 30° east of south
(S 30° E).

Note

Corporate Office : B-58, Goal Building, Budha Colony, Patna-1, Help line : 9334594165/66/67 Website : www.goalinstitute.org
Kinematics [ 64 ]

Daily Practice Paper - 4


1. A helicopter is flying south with a speed of 50 km/h. A train is moving with the same speed towards
east. The relative velocity of the helicopter as seen by the passengers in the train will be
502 km/h towards.
(1) north east (2) south east (3) north west (4) south west
2. Two particles are moving with velocities v1 and v2. Their relative velocity is the maximum when the
angle between their velocities is
(1) zero (2) /4 (3) /2 (4) 
3. A car is going eastwards with a velocity of 8m/s. To the passenger in the car, a train appears to be
moving northwards with a velocity 15 m/s. What is the actual velocity of the train ?
(1) 7m/s (2) 17m/s (3) 23 m/s (4) None of these
4. Two cyclists start at the same instant from the junction of two roads inclined at an angle 60° with
speed 12km/h and 8 km/hr respectively on each road. Find their relative velocity and their distance
from each other after 15 minutes.
5. A train is moving with a velocity of 30 km/h due east and a car is moving with a velocity of 40 km/
h due north. What is the velocity of car as appeared to the passengers in the train ?
6. The velocity of particle P due east is 4m/s and that of Q is 3 m/s due north. What is the velocity of
P w.r.t. to Q ?
7. To a man going east in a car with a velocity of 40 km/h, a train appears to move towards north with
a velocity of 403 km/h. What is the actual velocity and direction of motion of train ?
8. (i) Find out the magnitude of velocity and direction of motion of tree, bird and old man as seen by
boy
(ii) Find out the magnitude of velocity and direction of motion of tree, bird and boy as seen by old
man
(iii) Find out the magnitude of velocity and direction of motion of tree, boy and old man as seen by
bird.

s
m/
20 37º

Bird

Tree Boy Old man

10m/s

1m/s

9. A particle A is moving along a straight line with velocity 3 m/s and another particle B has a velocity
5m/s at an angle 37° to the path of A. Find the velocity of B relative to A.

Corporate Office : B-58, Goal Building, Budha Colony, Patna-1, Help line : 9334594165/66/67 Website : www.goalinstitute.org
Kinematics [ 65 ]

10. A ship is steaming due east at 12 ms–1. A woman runs across the deck at 5 ms–1 in a direction at
right angle to the direction of motion of the ship towards north. Calculate the velocity of the woman
relative to sea.

Answer Daily Practice Paper - 4


1. (4) 2. (4)
3. (2) 4. 10.58 km/h, 2.645 km
5. 50km/h, 37° west of north 6. 5m/s, 37° south of east
7. 80 km/h, 30° east of north
   
8. (i) 10 i, 6 i  12 j,  11 i

   
(ii) i,17 i  12 j  11 i

     
(iii) 16 i  12 j,  6 i  12 j,  17 i  12 j

9. 10 m/s at angle of (tan–1(3) to the path of A 10. 13m/s, tan(5/12) north of east.

Note

Corporate Office : B-58, Goal Building, Budha Colony, Patna-1, Help line : 9334594165/66/67 Website : www.goalinstitute.org
Kinematics [ 66 ]
5.12 RAIN PROBLEMS :
 
In these types of problems we again come across three terms vr ,v m and v rm .Here,


vr  velocity of rain

v m  velocity of man (it may be velocity of cyclist or velocity of motorist also)

and v r m  velocity of rain with respect to man

Here, vr m is the velocity of ran which appear to the man.
Ex. To man walking at the rate of 3km/h the rain appears to fall vertically. When he increases his
speed to 6km/h it appears to meet him at an angle of 45° with vertical. Find the speed of rain.
 
Soln. Let i and j be the unit vectors in horizontal and vertical directions respectively.
Let velocity of rain
  
v  a i b j …(i)
Then speed of rain will be

vr  a2  b2 …(ii)

 
In the first case v m  velocity of man  3 i
    
 v rm  v r  v m  (a  3) i  b j
It seems to be in vertical direction. Hence,
a – 3 = 0 or a = 3
 
In the second case v m  6 i
^
Vertical (j)
    
 v r m   a  6  i  b j  3 i  b j
This seems to be at 45° with vertical
Hence,
^
Therefore, from eq. (ii) speed of rain is Horizontal (i)

 2
v r  (3)2  (3)  3 2km / h

IMPORTANT POINTS–4
  
(a) Relative velocity of rain is given by v rm  vrg  v mg
(b) Be careful, you are using vector mathematics
(c) For inclination of umbrella, note it is parallel to direction of rain as seen by running man.

Corporate Office : B-58, Goal Building, Budha Colony, Patna-1, Help line : 9334594165/66/67 Website : www.goalinstitute.org
Kinematics [ 67 ]

Daily Practice Paper-5


1. Raindrops are falling vertically with a velocity 10m/s. To a cyclist moving on a straight road the rain
drops appear to be coming with a velocity of 20m/s. The velocity of cyclist is
(1) 10m/s (2) 10 3 m/s (3) 20 m/s (4) 20 3 m/s
2. To a man running at a speed of 5m/s, the rain drops appear to be falling at an angle of 45° with the
vertical. If the rain drops are actually falling downwards, then their velocity in m/sec is
(1) 5 (2) 5 4 (3) 5 2 (4) 4
3. Rain is falling vertically downwards with a velocity of 3km/h. A man walks in the rain with a velocity
of 4km/h. The rain drops will fall on the man with a velocity of
(1) 1km/h (2) 3km/h (3) 4km/h (4) 5km/h
4. A man walks in rain with a velocity of 5km/h. The raindrops strikes him at a angle 45° with the
horizontal. The downward velocity of the raindrops will be
(1) 5km/h (2) 4km/h (3) 3km/h (4) 1km/h
5. A car with a vertical wind shield moves along in rain storm at the speed of 40km/h. The raindrops
fall vertically with a terminal speed of 20m/s. The angle with the vertical at which the raindrop strike
the wind shield is
(1) tan–1(5/9) (2) tan–1(9/5) (3) tan–1(3/2) (4) tan–1(3)
6. Raindrops are falling vertically downwards at a speed of 5 km/h. Find at what angle a cyclist,
moving at a speed of 1 km/h should hold his umbrella to protect himself from the rain ?
7. To a man walking at the rate of 3 km/h the rain appears to fall vertically. When he increases his
speed to 4 km/h, the rain appears to meet him at an angle of 60° with the horizontal. Find the true
direction of the rain.
8. A car is moving on a road with a constant speed of 10m/s. The raindrops are falling towards car
with the velocity of 5 m/s at an angle 53° with the vertical. Find the velocity of raindrops with respect
to car.
9. A man standing on a road has to hold his umbrella at 30° with vertical to keep the rain away. He
throw the umbrella and starts running at 10km/h. He finds rain drops are hitting his head vertically.
Finding the speed of rain drops.
(i) w.r.t road (ii) w.r.t. running man
10. Raindrops are falling with a velocity 102 m/s making an angle of 45° with the vertical. The drops
appears to be falling vertically to a man running with constant speed. The velocity of raindrops
change such that the raindrops now appears to be falling vertically with 3 times the velocity it
appeared earlier to the same person running with same velocity. Find the angle (in degree) between
the initial and the final velocity vectors of the raindrops with respect to ground.

Answer Daily Practice Paper – 5


1. (2) 2. (1)
3. (4) 4. (1)
5. (1) 6. tan–1(1/5)
7. 30° 8. 205m/s
9. (i) 20 m/s, (ii) 103 m/s 10. 15°

Corporate Office : B-58, Goal Building, Budha Colony, Patna-1, Help line : 9334594165/66/67 Website : www.goalinstitute.org
Kinematics [ 68 ]
5.13 RIVER BOAT PROBLEMS :
In river boat problems we come across the following three terms :

vr  absolute velocity of river
 
v br  velocity of boatmanwithrespect toriver orvelocity of boatman in still water and v b  absolute
velocity of boatman.

A boatman starts from point A on one bank of river with velocity v br in the direction shown in

figure. River flowing along positive x-direction with velocity vr . Width of the river is d, then
  
v br  v b  v r
B
  
Therefore, v bx  v r  v br sin 
y
and v by  v br cos  d
Vbr
Now, time taken by the boatman to cross the river is  x

d d A 
t  Vr
v by v br cos 

d
or, t …(i)
v br cos 
Further, displacement along x-axis when he reaches on the other bank (also called drift) is
d
x  v bx t  (v r  v br sin )
v br cos 

d
or, x  (v r  v b sin) …(ii)
v br cos 
THREE SPECIAL CASES ARE :
B
(i) Condition when the boatman crosses the river in shortest
interval of time : From Eq. (i) we can see that time (t) will be

minimum when = 0°, i.e. the boatman should steer this boat Vr
perpendicular to the river current.
d A 
Also, t min  v as cos  1 Vr
br

(ii) Condition when the boatman wants to reach point B, i.e. at a point just opposite from
where he started
In this case, the drift (x) should be zero.
 x=0
d
or, v r  v br sin 0
v br cos 

vr v
or, sin or  sin–1 r 
v br  v br 

Corporate Office : B-58, Goal Building, Budha Colony, Patna-1, Help line : 9334594165/66/67 Website : www.goalinstitute.org
Kinematics [ 69 ]
(iii) Shortest path
x
x is minimum, where 0


or,

 d
 v br cos  
(v r  v br sin)  0

or, –v br cos 2– (v r – v brsin) (–sin) = 0


or, –v br + v rsin= 0

v
or,  sin–1 br 
 vr 
Ex. A man can row a boat with 4km/h in still water. If he is crossing a river where the current is 2km/h.
(1) In what direction will his boat be headed if he wants to reach a point on the other bank,
directly opposite to starting point ?
(2) If width of the river is 4km, how long will the man take to cross the river, with the condition in
part ?
(3) In what direction should he head the boat if he wants to cross the river is shortest time and
what is this minimum time ?
(4) How long will it take him to row 2 km up the stream and then back to his starting point ?
n
Sol . (1) Given, that vbr = 4km/h and vr = 2km/h


v

2 1
 sin–1 r   sin –1  sin–1  30
 v br  4 2 
Hence, the reach the point directly opposite to starting point he should head the boat at an
angle of 34°, with AB or 90° + 30 = 120° with the river flow.
(2) Time taken by the boatman to cross the river
d = width of river = 4km
v br = 4km/h and = 30°
4 2
 t · h
4cos30 3
(3) For shortest time = 0°
w 4
and t min  v cos 0   4  1 h
br

Hence, the should head his boat perpendicular to the river current for crossing the river in shortest
time and this shortest time is 1h.

vbr – vr vbr + vr
D C D C

(4) t = tCD + tDC


CD DC 2 2 1 4
or, t     1  h
vbr  vr vbr  vr 4  2 4  2 3 3

Corporate Office : B-58, Goal Building, Budha Colony, Patna-1, Help line : 9334594165/66/67 Website : www.goalinstitute.org
Kinematics [ 70 ]

IMPORTANT POINT–5
  
(a) This type of problems involve resultant of velocity v  v 1 v 2
(b) time to cross the river does not depends on the river flow (x–axis)

(c) To minimize the time to cross the river swimmer should swims perpendicular to the river flow.

Daily Practice Paper - 6


1. A river is flowing with a speed of 1km/h. A swimmer wants to go to point C starting from A. He
swims with a speed of 5km/h at an angle w.r.t. the river. If AB = BC = 400 m. Then the value of  is
1km/h B C


A
(1) 37° (2) 30° (3) 53° (4) 45°
2. A man who can swim at the rate of 2 km/h crosses a river to an exactly opposite point on the other
bank by swimming in a direction of 120° to the flow of the water in the river. The velocity of the
water current in km/h is
(1) 1 (2) 2 (3) 1/2 (3) 3/2
3. A swimmer crosses the river along the line making an angle of 45° with the direction of flow.
Velocity of the river water is 5m/s. Swimmer takes 6 seconds to cross the river of width 60m. The
velocity of the swimmer with respect to water will be

(1) 10 m/s (2) 12m/s (3) 5 3 m/s (4) 10 2 m/s


4. Two men ram and Gopal are competing to cross a river. Ram start from point A runs to bridge BC
and then crosses bridge maintaining a speed of 7 m/s. Gopal swims across to point C starting from
point A. His velocity in still water is 4m/s and river flows with velocity of 3 m/s. What is the time
taken by each person ?

C
400m
VR = 3m/s
A 300m B

5. A boat is moving towards east with velocity 4 m/s with respect to still water and river is flowing
towards north with velocity 2m/s and the wind is flowing towards north with velocity 6m/s. The
direction of the flag blown over by the wind horizontal on the boat is
(1) north-west (2) south-west (3) tan1(1/2) with east (4) north
6. A river is flowing from west to east at a speed of 5 meters per minute. A man on the south bank of
the river, capable of swimming at 10 meters per minute in still water, wants to swim across the river
in shortest time. He should swim in a direction
(1) due north (2) 30° east of north (3) 30° north of west (4) 60° east of north

Corporate Office : B-58, Goal Building, Budha Colony, Patna-1, Help line : 9334594165/66/67 Website : www.goalinstitute.org

Note
Kinematics [ 71 ]
7. A swimmer crosses a flowing stream of breadth b to and fro in time T1. The time taken to cover the
same distance up and down the stream is T2. If T3 is the time the swimmer would take to swim a
distance 2b in still water, then
(1) T3 = T1 + T2 (2) T12 = T2 T3 (3) T22 = T1T3 (4) T32 = T1T2
8. A boat is rowed with a velocity of 6km/h straight across a river which flows at the rate of 2km/h. If
its breadth be 300 meters, find how far down the river the boat will reach the opposite bank below
the point at which it was originally directed.
9. A swimmer starts to swim from point A to cross a river. He wants to reach point B on the opposite
side of the river. The line AB makes an angle 60° with the river flows as shown. The velocity of the
swimmer in still water is same as that of the water.
(1) In what direction should he try to direct his velocity ? Calculate angle between his velocity and
river velocity
(2) Find the ratio of the time taken to cross the river in this situation to the minimum time in which
he can cross this river.

60º
A

10. If a man in a boat crosses a river from point A and rows perpendicular to the banks then he will
reach point C lying at a distance 120 m downstream from point B in 10 minutes. If the man heads
at a certain angle to the straight line AB, (perpendicular to banks) against the current he will reach
point B after 12.5 minutes. Find the width of the river d, velocity of boat u relative to river the speed
of the current v and angle  Assume the velocity of boat relative to the water to be constant and
same in both cases.
B C

 v
A

Answer Daily Practice Paper-6


1. (3) 2. (1)
3. (4) 4. 100 seconds
5. (1) 6. (1)
7. (2) 8. 100m
9. (1) 120° (2) 2/3

10. v = 1/5 m/s,  = 37°, u = 1/3 m/s width 200 m

Corporate Office : B-58, Goal Building, Budha Colony, Patna-1, Help line : 9334594165/66/67 Website : www.goalinstitute.org
Kinematics [ 72 ]
SOME ADDITIONAL PROBLEM BASED ON RELATIVE VELOCITY
Ex. Two trains along the same straight rails moving with constant speed 60 km/hr respectively toward
each other. If at time t = 0, the distance between them is 90 km, the time when they collide is
(1) 1 hr (2) 2 hr (3) 3 hr (4) 4 hr
Soln. (1) The relative velocity vrel = 60 – (–30) = 90 km/hr.
srel. 90
Distance between the train Srel. = 90km,  Time when they collide  v   1hr.
rel. 90
Ex. Two cars are moving in the same direction with the same speed 30 km/hr. They are separated by
a distance of 5 km, the speed of a car moving in the opposite direction if it meets these two cars at
an interval of 4 minutes, will be
(1) 40 km/hr (2) 45 km/hr (3) 30 km/hr (4) 15 km/hr
n
Sol . (2) The two car (say A and B) are moving with same velocity, the relative velocity of one (say B)
  
with respect to the other A, v BA  v B  v A  v  v  0 .
So the relative separation between them (= 5 km) always remains the same.

Now if the velocity of car (say C) moving in opposite direction to A and B, is v C relative to ground
  
then the velocity of car C relative to A and B will be v rel.  v C  v

But as v is opposite to vC so v rel  v c  (30)  (v C  30)km / hr.

d 4 5
So, the time taken by it to cross the cars A and B t  v  
60 v C  30  v C = 45km/hr
rel
Ex. A steam boat goes across a lake and comes back (1) On a quite day when the water is still and (2)
On a rough day when there is uniform current so as to help the journey onward and to impede the
journey back. If the speed of the launch on both days was same, in which case it will complete the
journey in lesser time
(1) Case (1) (2) Case (2)
(3) Same in both (4) Nothing can be predicted
n
Sol . (2) If the breadth of the lake is l and velocity of boat is vb. Time in going and coming back on a
quite day
l l 2l
tQ    .....(i)
vb vb vb
Now if v a is the velocity of air- current then time taken in going across the lake,
l
t1  [as current helps the motion]
vb  va
l
and time taken in coming back t 2  [as current opposes the motion]
vb  v a
2l
So tR =t1 + t2  .....(ii)
vb [1  (v a / v b )2 ]

Corporate Office : B-58, Goal Building, Budha Colony, Patna-1, Help line : 9334594165/66/67 Website : www.goalinstitute.org
Kinematics [ 73 ]

tR 1 v 2a
From equation (i) and (ii)   1 [as 1   1] i.e. tR > tQ
tQ [1  (v a / vb )2 ] vb2
i.e. time taken to complete the journey on quite day is lesser than that on rough day.
Ex. A man standing on a road hold his umbrella at 30° with the vertical to keep the rain away. He
throws the umbrella and starts running at 10 km/hr. He finds that raindrops are hitting his head
vertically, the speed of raindrops with respect to the road will be
(1) 10 km/hr (2) 20 km/hr (3) 30 km/hr (4) 40 km/hr
n
Sol . (2) When the man is at rest w.r.t. the ground, the rain comes to him at an angle 30° with the
vertical. This is the direction of the velocity of raindrops with respect to the ground.

Here v rg  velocity of rain with respect to the ground

v mg  velocity of the man with respect to the ground.

and v rm  velocity of the rain with respect to the man,

v mg

30°

v rm v rg

  
We have v rg  v rm  v mg …(i)

Taking horizontal components equation (i) gives v r g sin 30  vm g  10 km / hr

10
or vr g   20 km / hr
sin30
Ex. In the above problem, the speed of raindrops w.r.t. the moving man, will be
(1) 10 / 2 km / h (2) 5 km/h (3) 10 3 km / h (4) 5 / 3 km / h

3
Soln. (3) Taking vertical components equation (i) gives v rg cos 30  v rm  20  10 3 km / hr
2
Ex. Two cars are moving in the same direction with a speed of 30 km/h. They are separated from
each other by 5 km. Third car moving in the opposite direction meets the two cars after an interval
of 4 minutes. What is the speed of the third car
(1) 30 km/h (2) 35 km/h (3) 40 km/h (4) 45 km/h
Soln. (4) Let v be the velocity of third car, then relative velocity of third car w.r.t. the either car is
v – (– 30) = (v + 30) km/h.
Now (v + 30) × (4/60) = 5  v = 45 km/h
Ex. To a person, going eastward in a car with a velocity of 25 km/hr, a train appears to move towards
north with a velocity of 25 3 km/hr. The actual velocity of the train will be

(1) 25 km/hr (2) 50 km/hr (3) 5 km/hr (4) 5 3 km/hr


Corporate Office : B-58, Goal Building, Budha Colony, Patna-1, Help line : 9334594165/66/67 Website : www.goalinstitute.org
Kinematics [ 74 ]

Soln. (1) v T  v TC
2 2
 vC

= (25 3) 2  (25)2

= 1875  625
= 2500 = 50 km/hr
Ex. A boat is moving with a velocity 3i + 4j with respect to ground. The water in the river is moving with
a velocity – 3i – 4j with respect to ground. The relative velocity of the boat with respect to water is
(1) 8j (2) – 6i – 8j (3) 6i +8j (4) 5 2
Soln. (3) Relative velocity = (3i + 4j) – (– 3i – 4j) = 6i + 8j
5.14 AIRCRAFT WIND PROBLEMS :
 
This is similar to river boat problem, the only difference is that v br is replaced by v aw (velocity of
 
aircraft with respect to wind or velocity of aircraft in still air), v r is replaced by v w (velocity of wind)
    
and v b is replaced by v a (absolute velocity of aircraft). Further, v a  v aw  v w
Ex. An aircraft flies at 400 km/h in still air. A wind of 200 2km/h is blowing from the south. The pilot
wishes to travel from A to a point B north east of A. Find the direction he must setter and time of his
journey if AB = 1000 km.
Soln. Given that vw = 200 2km/h

 
aaw = 400km/h and v a should be along AB or in north-east direction. Thus, the direction of v aw
 
should be such as the resultant of v w and v aw is along ASB or north-east direction.

Let vaw makes an angle with AB as shown in the figure. Applying sine law in triangle ABC, we get
AC BC

sin45 sin

or, sin   BC
AC
sin45

 200 2  1 1
  
 400  2 2
 = 30°
Therefore, the pilot should steer in a direction at an angle of (45° – ) or 75° from north towards
east.

Corporate Office : B-58, Goal Building, Budha Colony, Patna-1, Help line : 9334594165/66/67 Website : www.goalinstitute.org
Kinematics [ 75 ]


va 400
Further 
sin(180  45 30 ) sin45

 sin105  km
or, va  × (400)
sin45 h

 cos 15  km
  (400)
 sin 45  h

 0.9659  km
  (400)
 0.707  h
= 546.47 km/h
 The time of journey from A to B is

AB 1000
t   h
546.47
va

t = 1.83 h

Relative velocity of satellite : If a satellite is moving in equatorial plane with velocity v s and a

point on the surface of earth with v e relative to the centre of earth, the velocity of satellite relative
to the surface of earth
  
v se  v s  v e

So if the satellite moves form west to east (in the direction of rotation of earth on its axis) its
velocity relative to earth’s surface will be v se  v s  v e
And if the satellite moves from east to west, i.e., opposite to the motion of earth,
v se  v s  (v e )  v s  v e

Additional
1. PROJECTILE MOTION ON INCLINED PLANE :
(a) Range of projectile on inclined plane :

2u2 sin(   ).cos 


R 2 y
gcos 
(b) Time of flight : u A

2u sin(  ) R
T
g cos   Rsin

O Rsin X

Corporate Office : B-58, Goal Building, Budha Colony, Patna-1, Help line : 9334594165/66/67 Website : www.goalinstitute.org
Kinematics [ 76 ]
(c) for maximum range :
Let us calculate so that range OA is maximum

2u2 sin(   ) cos  u2  sin(2  )  sin 


R 2 
gcos  g cos2 


For maximum range, sin(2 – ) must be maximum. Hence 2– = so that R is maximum.
2

  u2 (1  sin )
 R is maximum for    the maximum value of Rmax  2
4 2 gcos 

u2
 Rmax 
g(1 sin)
2. PROBLEMS BASED ON VELOCITY OF APPROACH :
Ex. Three particles A, B and C are situated at the vertices of an equilateral traingle ABC of side d at t = 0.
Each of the particles moves with constant speed v. A always has its velocity along AB, B along BC and
C along CA. At what time will the particles meet each other ?
n
Sol . The motion of the particles is roughly sketched in figure (3-W15). By symmetry they will meet at the
centroid O of the triangle. At any instant the particles will form an equilateral triangle ABC withthe same
centroid O. Concentrate on the motion of any one particles, say A. At any instant its velocity makes
angle 30° with AO.
The component of this velocity along AO is v cos30°. This component is the rate of decrease of the
distance AO. Initially,
2
AO 
2 2 d
3
d 
2 
d
 .
3
A

B C

Therefore, the time taken for AO to become zero


d/ 3 2d 2d
   .
b cos30 3v  3 3v
Alternative : Velocity of A is along AB. The velocity of B is along BC. Its component along BA is v cos
60° = v/2. Thus, the sepeartion AB decreases at the rate
v 3v
v  .
2 2

Since this rate is constant, the time taken in reudcing the seperation AB from d to zero is t  d  2d .
v 3v
3
2
Corporate Office : B-58, Goal Building, Budha Colony, Patna-1, Help line : 9334594165/66/67 Website : www.goalinstitute.org
Kinematics [ 77 ]
Ex. Six particles situated at the corners of a regular hexagon of side a move at a cosntant speed v. Each
particle maintains a direction towards the particle at the next corner. Calculate the time the particles
will take to meet each other.
Soln. The relative velocity of a particle with respect to other is
u
vr  u E D
2

u
vr  F C
2
Time taken by particle to meet
60º
d 2a A B
t 
vr u
Ex. A car is moving at constant speed of 40 km/h along a straight road which heads towards a large vertical
wall and makes a sharp 90° turn by the side of the wall. A fly flying at a constant speed of 100 km/h,
starts from the wall towards the car at an instant when the car is 20 km away, flies until it reaches the
glasspane of the car and returns to the wall at the same speed. It continues to fly between the car and
the wall till the car makes the 90° turn. (a) What is the total distnace the fly has travelled during this
period? How many trips has it made between the car and the wall ?

20km 1
Soln. (a) The time taken by the car to cover 20km before the turn is  h. The fly moves at a
40km /h 2
constant speed of 100 km/h during this time. Hence the total distance covered by it is
km 1
100  h  50km.
h 2
(b) Suppose the car is at a distance x away (at A) when the fly is at the wall (at O). The fly hits the
glasspane at B, taking a time t. Then
AB = (40 km/h)t,
and OB = (100 km/h)t
Thus, x = AB + OB
= (140 km/h)t A B C O

x 5
or, t , or OB  x
140km/h 7
The fly returns to the wall and during this period the car moves the distance BC. The time taken by the
fly in this return path is.

100km/h
5x/ 7
 140km
x
/h

Thus, BC  40x  2 x.
140 7

Or, OC  OB  BC  3 x.
7

Corporate Office : B-58, Goal Building, Budha Colony, Patna-1, Help line : 9334594165/66/67 Website : www.goalinstitute.org
Kinematics [ 78 ]

3
If at the beginning of the round trip (wall to the car and back) the car is at distance x away, it is x away
7
when tne next trip again starts.
Distance of the car at the beginning of the 1st trip = 2km.

3
Distance of the car at the beginning of the 2nd trip   20km
7
2

Distance of the car at the beginning of the 3rd trip  32 20km
3

Distance of the car at the beginning of the 4th trip  


3
7
20km

n1

Distance of the car at the beginning of the nth trip   


3
20km.
7
Trips will go on till the car reaches the turn that is the distance reduces to zero. This will be the case
when n becomes infinity. Hence the fly makes an infinite number of trips before the car takes the turn.

Note

Corporate Office : B-58, Goal Building, Budha Colony, Patna-1, Help line : 9334594165/66/67 Website : www.goalinstitute.org
Kinematics [ 79 ]

Exercise-2
MOTION IN ONE DIMENSIONS (‘0’ level)
1. An athlete completes one round of a circular track of radius R in 40 seconds. What will be his
displacement at the end of 2 minutes 20 seconds
(1) Zero (2) 2R (3) 2r (4) 7r
2. A boy stops after travelling 3 km towards east and then goes 4 km towards north along a plane
road. The resultant displacement of the boy is
(1) 7 km (2) 4 km (3) 5 km (4) 15 km
3. A 150 m long train is moving with a uniform velocity of 45 km/h. The time taken by the train to cross
a bridge of length 850 meters is
(1) 56 sec (2) 68 sec (3) 80 sec (4) 92 sec
4. A particle moves along x-axis in such a way that its coordinate x varies with time t according to the
equation x = (2 – 5t + 6t2)m. The initial velocity of the particle is
(1) –5 m/s (2) 6 m/s (3) – 3 m/s (4) 3 m/s
5. A car travels a distance S on a straight road in two hours and then returns to the starting point in
the next three hours. Its average velocity is
(1) S/5 (2) 2S/5 (3) S/2 + S/3 (4) None of the above
6. When a particle moves with uniform velocity, which of the following relations are correct
(I) Average speed = average velocity
(II) Instantaneous speed = instantaneous velocity
(III) Distance covered = magnitude of displacement
(1) I, II, III (2) I, II (3) II, III (4) I, III
7. When a particle moves with variable velocity, which of the following statements are not correct
(I) Average speed = average velocity
(II) Instantaneous speed = instantaneous velocity
(III) Distance covered = magnitude of displacement
(1) I, II,III (2) I, II (3) II, III (4) I, III
8. A particle moves along the sides AB, BC, CD of a square of side 25 m with a velocity of 15m/s Its
average velocity is
C B

D A

(1) 15 ms –1 (2) 10 ms –1 (3) 7.5 ms –1 (4) 5 ms –1


9. A body has speed V, 2V and 3V in first 1/3 of distance S, seconds 1/3 of S and third 1/3 of S
respectively. Its average speed will be
18 11
(1) V (2) 2V (3) V (4) V
11 18
Corporate Office : B-58, Goal Building, Budha Colony, Patna-1, Help line : 9334594165/66/67 Website : www.goalinstitute.org
Kinematics [ 80 ]
10. A particle moves along a straight line such that its displacement at any time t is given by
S  t3  6t 2  3t  4 metres .The velocity when the acceleration is zero is
(1) 3 ms –1 (2) –12 ms–1 (3) 42 ms –1 (4) –9 ms –1
11. The velocity of a body depends on time according to the equation v = 20 + 0.1 t2. The body is
undergoing
(1) Uniform acceleration (2) Uniform retardation
(3) Non-uniform acceleration (4) Zero acceleration
12. The displacement of a body is given to be proportional to the cube of time elapsed. The magnitude
of the acceleration of the body is
(1) Increasing with time (2) Decreasing with time
(3) Constant but not zero (4) Zero
13. The correct statement from the following is
(1) A body having zero velocity will not necessarily have zero acceleration
(2) A body having zero velocity will necessarily have zero acceleration
(3) A body having uniform speed can have only uniform acceleration
(4) A body having non-uniform velocity will have zero acceleration
14. What is the angle between instantaneous displacement and acceleration during the retarded motion
 
(1) Zero (2) (3) (4) 
4 2
15. Which of the following displacement time graphs is not possible
x x x x

(1) (2) (3) (4)


O t
O t O t O t
16. An object is moving with a uniform acceleration which is parallel to its instantaneous direction of
motion. The displacement (s) – velocity (v) graph of this object is
s s s s

(1) (2) (3) (4)


v v v v

17. A car accelerates from rest at a constant rate  for some time, after which it decelerates at a
constant rate  and comes to rest. If the total time elapsed in t, then the maximum velocity acquired
by the car is
  2 2    2 2  ( )t t
(1)   t (2)   t (3)  (4)  
   
18. A rocket is projected vertically upwards, whose velocity-time graph is shown in fig. The maximum
height reached by the rocket is A
Velocity (m/sec)

1000
(1) 1 km
(2) 10 km C B
0
(3) 20 km 20 40 60 80 100120 140
(4) 60 km Time (sec)

Corporate Office : B-58, Goal Building, Budha Colony, Patna-1, Help line : 9334594165/66/67 Website : www.goalinstitute.org
Kinematics [ 81 ]

19. In the above problem the mean velocity of rocket in reaching the maximum height will be
(1) 100 m/s (2) 50 m/s (3) 500 m/s (4) 25/3 m/s
20. In the above problem the acceleration of rocket will be
(1) 50 m/s2 (2) 100 m/s 2 (3) 500 m/s 2 (4) 250 m/s 2
21. The figure shows the velocity of a particle plotted against time t

O t
T 2T

(1) The displacement of the particle is zero


(2) The particle changes its direction of motion at some point
(3) The initial and final speeds of the particle are same
(4) All of the above statements are correct
22. The v – t plot of a moving object is shown in the figure. The average velocity of the object during
the first 10 seconds is

5
-1
Velocity (ms )

Time (sec)
0
5 10

–5

(1) 0 (2) 2.5 ms-1 (3) 5 ms-1 (4) 2 ms-1


23. A constant force acts on a body of mass 0.9 kg at rest for 10s. If the body moves a distance of
250 m, the magnitude of the force is
(1) 3N (2) 3.5N (3) 4.0N (4) 4.5N
24. If a train travelling at 72 kmph to be brought to rest in distance of 200 metres, then its retardation
should be
(1) 20 ms–2 (2) 10 ms–2 (3) 2 ms –2 (4) 1 ms –2
25. A car moving with a speed of 40 km/h can be stopped by applying brakes after at least 2 m. If the
same car is moving with a speed of 80 km/h what is the minimum stopping distance
(1) 8 m (2) 2 m (3) 4 m (4) 6 m
26. A particle moves along a straight line path. After some time it comes to rest. The motion is with
constant acceleration whose direction with respect to the direction of velocity is
(1) Positive throughout motion (2) Negative throughout motion
(3) First positive then negative (4) First negative then positive
–1
27. A bus is moving with a velocity 10 ms on a straight road. A scooterist wishes to overtake the bus
in 100 s. If, the bus is at a distance of 1 km from the scooterist, with what velocity should the
scooterist chase the bus
(1) 50 ms –1 (2) 40 ms –1 (3) 30 ms –1 (4) 20 ms –1

Corporate Office : B-58, Goal Building, Budha Colony, Patna-1, Help line : 9334594165/66/67 Website : www.goalinstitute.org
Kinematics [ 82 ]
28. The velocity acquired by a body moving with uniform acceleration is 30 m/s in 2s seconds and
60 m/s in four seconds. The initial velocity is
(1) 4 ms –1 (2) 0 ms –1 (3) 2 ms –1 (4) 10 ms–1
29. An engine of a train moving with uniform acceleration passes an electric pole with velocity u and
the last compartment with velocity v. The middle point of the train passes past the same pole with
a velocity of

v 2 u2 v 2  u2 u2  v 2 u v
(1) (2) (3) (4)
2 2 2 2
30. A uniformly accelerated body passes two points P and Q with speeds of 10m/s and 20m/s
respectively. If O is mid-point of P and Q then speed at O will be
(1) 15.0 m/s (2) 15.8 m/s (3) 16.5 m/s (4) 14.2 m/s
31. A man throws a ball vertically upward and it rises through 20 m and returns to his hands. What was
the initial velocity u of the ball and for how much time (T) it remains in the air [g 10m/ s 2 ]
(1) u = 10 m/s, T = 2s (2) u = 10 m/s, T = 4s
(3) u = 20 m/s, T = 2s (4) u = 20 m/s, T = 4s
32. A balloon starts rising from the ground with an acceleration of 1.25 m/s2 after 8s a stone is released
from the balloon. The stone will (g = 10 m/s2)
(1) Reach the ground in 4 second
(2) Begin to move down after being released
(3) Have a displacement of 50 m
(4) Cover a distance of 40 m in reaching the ground
33. A body thrown vertically upwards with an initial velocity v reaches maximum height in 6 seconds.
The ratio of the distances travelled by the body in the first second and the seventh second is
(1) 1 : 1 (2) 11 : 1 (3) 1 : 2 (4) 1 : 11
34. Time taken by an object to reach the height of h1 and h2 is respectively t1 and t2 then the ratio of
t1 to t2 is
(1) h1 :h2 (2) h1 : h2 (3) h1 :2h2 (4) 2h1 : h2
35. The time taken by a block of wood (initially at rest) to slide down a smooth inclined plane 9.8 m
long (angle of inclination is 30° ) is
1
(1) sec (2) 2 sec (3) 4 sec (4) 1 sec
2
36. A stone is thrown with an initial speed of 4.9 m/s from a bridge in vertically upward direction. It falls
down in water after 2 sec. The height of the bridge is
(1) 4.9 m (2) 9.8 m (3) 19.8 m (4) 24.7 m
37. A ball is dropped downwards. After 1 second another ball is dropped downwards from the same
point. What is the distance between them after 3 seconds (g = 10 m/s2)
(1) 25 m (2) 20 m (3) 50 m (4) 9.8 m
38. A body dropped from a height h with an initial speed zero, strikes the ground with a velocity
3 km/h Another body of same mass is thrown downward from the same height h with an initial
speed 4km/h. Find the final velocity of second body with which it strikes the ground
(1) 3 km/h (2) 4 km/h (3) 5 km/h (4) 12 km/h
Corporate Office : B-58, Goal Building, Budha Colony, Patna-1, Help line : 9334594165/66/67 Website : www.goalinstitute.org
Kinematics [ 83 ]
39. A body is projected up with a speed 'u' and the time taken by it is T to reach the maximum height H.
Pick out the correct statement
(1) It reaches H/2 in T/2 sec (2) Its velocity is u/4 in T/2sec
(3) Its velocity is u/2 at T/2 sec (4) Same velocity at 2T
40. P, Q and R are three balloons ascending with velocities u, 4u and 8u respectively. If stones of the
same mass be dropped from each, when they are at the same height, then
(1) They reach the ground at the same time (2) Stone from P reaches the ground first
(3) Stone from R reaches the ground first (4) Stone from Q reaches the ground first
41. A particle is dropped vertically from rest from a height. The time taken by it to fall through successive
distances of 1m each will then be

(1) All equal, being equal to 2/g second


(2) In the ratio of square roots of the integers 1, 2, 3, ….
(3) In the ratio of the difference in the square roots of the integers
i.e. 1,( 2  1),( 3  2),( 4  3).......

1 1 1 1
(4) In the ratio of the reciprocal of the square roots of the integers i.e. , , ,
1 2 3 4
42. A rocket is fired upward from the earth's surface such that it creates an acceleration of
19.6 m/sec2. If after 5 sec its engine is switched off, the maximum height of the rocket from
earth's surface would be
(1) 245 m (2) 490 m (3) 980 m (4) 735 m
43. A man in a balloon rising vertically with an acceleration of 4.9 m/s2 releases a ball 2 sec after the
balloon is let go from the ground. The greatest height above the ground reached by the ball is
(g  9.8m/sec 2 )
(1) 14.7 m (2) 19.6 m (3) 9.8 m (4) 24.5 m
44. A body is slipping from an inclined plane of height h and length l. If the angle of inclination is  the
time taken by the body to come from the top to the bottom of this inclined plane is

2h 2l 1 2h 2h
(1) g (2) g (3) sin g (4) sin g

45. A frictionless wire AB is fixed on a sphere of radius R. A very small spherical ball slips on this wire.
The time taken by this ball to slip from A to B is
A

O
B R

2 gR cos  R gR
(1) gcos (2) 2 gR. (3) 2 (4)
g g gcos

Corporate Office : B-58, Goal Building, Budha Colony, Patna-1, Help line : 9334594165/66/67 Website : www.goalinstitute.org
Kinematics [ 84 ]
46. A body is released from the top of a tower of height h. It takes t sec to reach the ground. Where will
t
be the ball after time
2
h
(1) At from the ground
2
2h
(2) At from the ground
3
(3) Depends upon mass and volume of the body
3h
(4) At from the ground
4
47. A body is thrown vertically upwards. If air resistance is to be taken into account, then the time
during which the body rises is
(1) Equal to the time of fall (2) Less than the time of fall
(3) Greater than the time of fall (4) Twice the time of fall
48. A body falls freely from rest. It covers as much distance in the last second of its motion as covered
in the first three seconds. The body has fallen for a time of
(1) 3 s (2) 5 s (3) 7 s (4) 9 s
49. A ball is dropped on the floor from a height of 10 m. It rebounds to a height 2.5 m. If the ball is in
contact with the floor for 0.01 sec, the average acceleration during contact is
(1) 2100 m/s2 downwards (2) 2100 m/s2 upwards
2
(3) 1400 m/s (4) 700 m/s 2
50. Two particles one 0.98 m vertically above the other are released simultaneously. They fall under
gravity The separation between the two particles after 2 s will be
(1) 0.49 m (2) 4.9 m (3) 0.98 m (4) 19.6 m
51. Two balls are dropped from different heights. One ball is dropped 2 sec after the other ball. If both
balls reach the ground simultaneously after 5 sec of dropping the first ball the difference of initial
heights of the two balls will be
(1) 58.8 m (2) 78.4 m (3) 98 m (4) 117.6 m
52. A balloon is moving upwards with a constant velocity of 5m/s a stone is dropped from it. If at the
moment of dropping the balloon is at height of 50 m, then when the stone will hit the ground, at that
time the height of the balloon will be (g  10m/s 2 )
(1) 68.5 m (2) 63.5 m (3) 75.5 m (4) 88.7 m
53. A stone thrown upwards with a velocity u reaches upto a height h. If the initial velocity is 2u the
height attained would be
(1) 2 h (2) 4 h (3) 8 h (4) 16 h
54. A person throws balls into the air one after the other at an interval of one second. The next ball is thrown
when the velocity of the ball thrown earlier is zero. To what height the ball rise (Take g = 10 m/s2)
(1) 5 m (2) 10 m (3) 25 m (4) 40 m
55. A body is projected vertically up with a velocity v and after some time it returns to the point from
which it was projected. The average velocity and average speed of the body for the total time of
flight are
 
(1) v / 2 and v /2 (2) 0 and v /2 (3) 0 and 0 (4) v / 2 and 0
Corporate Office : B-58, Goal Building, Budha Colony, Patna-1, Help line : 9334594165/66/67 Website : www.goalinstitute.org
Kinematics [ 85 ]
56. Two balls A and B are simultaneously thrown. A is thrown from ground level with a velocity of
20m/s in the upward direction and B is thrown from a height of 40 m in the downward direction with
same velocity. Where will the two balls meet from the ground level
(1) 15 m (2) 25 m (3) 35 m (4) 45 m
57. A stone is dropped from a height h. Simultaneously, another stone is thrown up from the ground
which reaches a height 4 h. The two stones cross each other after time
h h
(1) 8g (2) 8gh (3) 2gh (4) 2g
58. A body is released from a height towards the ground level. Just after one second another body is
released from same height. The distance between two bodies just after two seconds after the
release of second body will be
(1) 4.9 m (2) 9.8 m (3) 19.6 m (4) 24.5 m
59. A stone falls from the top of the tower in 8 sec. How much time will it take to cover the first quarter
of the distance starting from the top
(1) 4 sec (2) 2 sec (3) 1 sec (4) None of these
60. Three particles A, B and C are thrown from the top of a tower with the same speed. A is thrown
straight up, B is thrown straight down and C is thrown horizontally. They hit the ground with speeds,
VA, VB and VC respectively, then
(1) v A  vB  vC (2) v A  vB  vC (3) v A  vB  vC (4) v A  vB  vC

Answer Exercise - 2
1. (2) 2. (3) 3. (3) 4. (1) 5. (4) 6. (1)
7. (4) 8. (4) 9. (3) 10. (4) 11. (3) 12. (1)
13. (1) 14. (4) 15. (1) 16. (3) 17. (4) 18. (4)
19. (3) 20. (1) 21. (4) 22. (1) 23. (4) 24. (4)
25. (1) 26. (2) 27. (4) 28. (2) 29. (2) 30. (2)
31. (4) 32. (1) 33. (2) 34. (2) 35. (2) 36. (2)
37. (1) 38. (3) 39. (3) 40. (2) 41. (3) 42. (4)
43. (1) 44. (3) 45. (3) 46. (4) 47. (2) 48. (2)
49. (2) 50. (3) 51. (2) 52. (1) 53. (2) 54. (1)
55. (2) 56. (1) 57. (1) 58. (4) 59. (1) 60. (1)

Note

Corporate Office : B-58, Goal Building, Budha Colony, Patna-1, Help line : 9334594165/66/67 Website : www.goalinstitute.org
Kinematics [ 86 ]

Exercise - 3
Motion in one dimension ‘A’ Level
1. A particle moves along a circular arc of radius R making an angle of  at centre. The magnitude of
displacement is
(1) 2 R sin /2 (2) 2R sin  (3) R sin  / 2 (4) R sin 
2. A particle moving in a straight line covers half the distance with speed of 3 m/s. The other half of
the distance is covered in two equal time intervals with speed of 4.5m/s and 7.5m/s respectively.
The average speed of the particle during this motion is

(1) 4.0 m/s (2) 5.0 m/s (3) 5.5 m/s (4) 4.8 m/s
3. If the body covers one-third distance at speed x1, next one third at speed x2 and last one third at
speed x 3, then average speed will be

x 1x 2  x 2 x 3  x 3 x 1 x1  x2  x3
(1) x1  x 2  x 3 (2)
3

x1 x 2 x 3 3x1 x 2 x3
(3) x 1x 2  x 2 x 3  x 3 x 1 (4) x1 x 2 + x 2 x 3 + x3 x 1

k
4. The displacement of the particle varies with time according to the relation x  (1e –bt ) . Then the
b
velocity of the particle is

k
(1) k(e bt ) (2) (3) kbe bt (4) None of these
b2e bt

5. The displacement of a particle is given by x  t 1. Which of the following statements about its
velocity is true
(1) It is zero (2) It is constant but not zero
(3) It increases with time (4) It decreases with time
6. The acceleration of a particle starting from rest, varies with time according to the relation
A = – a2 sint. The displacement of this particle at a time t will be

1
(1) 
2
a 2 sint t 2 (2) a  sin  t

(3) a  cos  t (4) a sin  t


7. If the velocity of a particle is (10 + 2t2) m/s, then the average acceleration of the particle between
2s and 5s is
(1) 2m/s 2 (2) 4m/s 2 (3) 12m/s 2 (4) 14m/s 2
8. A particle starts from rest, accelerates at 2 m/s2 for 10s and then goes for constant speed for 30s
and then decelerates at 4 m/s2 till it stops. What is the distance travelled by it
(1) 750 m (2) 800 m (3) 700 m (4) 850 m
Corporate Office : B-58, Goal Building, Budha Colony, Patna-1, Help line : 9334594165/66/67 Website : www.goalinstitute.org
Kinematics [ 87 ]
9. The graph below shows the velocity versus time graph for a body.
v

Which of the following graphs represents the corresponding acceleration versus time graphs

a a a a

(1) t (2) t (3) t (4) t

10. The acceleration-time graph for a body is shown in the following graph.
a

Which of the following graphs would probably represent the velocity of the body plotted against
time

v v
v v

(1) (2) (3) (4)


t t
t t

11. A particle is moving in such a way that its displacement is related with time by the equation
x  (10  4t  6t 2 ) m. The diagram showing variation of velocity of particle with time is
Velocity
Velocity

(1) Time (2) Time


Velocity

Velocity

(3) Time
(4)
Time

Corporate Office : B-58, Goal Building, Budha Colony, Patna-1, Help line : 9334594165/66/67 Website : www.goalinstitute.org
Kinematics [ 88 ]
12. A point moves with uniform acceleration v1, v 2 and v 3 denote the average velocities in the three
successive intervals of time t1, t2 and t3 . Which of the following relations is correct
(1) (v 1  v 2 ):(v 2  v 3 )  (t1  t2 ):(t 2  t 3 ) (2) (v 1  v 2 ):(v 2  v 3 )  (t1 t 2 ):(t 2  t 3 )
(3) (v 1  v 2 ):(v 2  v 3 )  (t 1  t 2 ):(t1  t3 ) (4) (v 1  v 2 ):(v 2  v 3 )  (t1  t 2 ):(t 2  t 3 )
13. A body is moving from rest under constant acceleration and let S1 be the displacement in the first
(p –1)sec and S2 be the displacement in the first p sec. The displacement in (p 2  p  1)th sec will be
(1) S1  S2 (2) S1S2 (3) S1  S2 (4) S1 /S2
14. A thief is running away on a straight road in jeep moving with a speed of 9 m/s. A police man
chases him on a motor cycle moving at a speed of 10m/s. If the instantaneous separation of the
jeep from the motorcycle is 100 m, how long will it take for the police to catch the thief
(1) 1 s (2) 19 s (3) 90 s (4) 100 s
15. A car A is travelling on a straight level road with a uniform speed of 60 km/h. It is followed by
another car B which is moving with a speed of 70 km/h. When the distance between them is 2.5
km, the car B is given a deceleration of 20 km/h2. After how much time will B catch up with A
(1) 1 hr (2) 1/2 hr (3) 1/4 hr (4) 1/8 hr
16. Two cars A and B are travelling in the same the direction with velocities v1 and v 2 . When the car
A is at a distance d ahead of the car B, the driver of the car A applies the brake producing a uniform
retardation a there will be no collision when
(v1  v 2 )2 (v 21  v 22 ) (v 1  v 2 )2 v 12  v 22
(1) d (2) d (3) d (4) d
2a 2a 2a 2a
17. Two trains one of length 100 m and another of length 125 m, are moving in mutually opposite
directions along parallel lines, meet each other, each with speed 10 m/s . If their acceleration are
0.3m/s 2 and 0.2m/s2 respectively, then the time they take to pass each other will be
(1) 5 s (2) 10 s (3) 15 s (4) 20 s
18. If the distances covered by an accelerated body during the l th, mth and nth seconds are a, b and c
respectively, then the correct relation is
(1) a(m n) b(n l )  c(l  m)  0 (2) l (b  c) m(c  a) n(a  b)  0
(3) al bm  cn  0 (4) None of these is true
19. Two trains, one travelling at 90 m/s and the other travelling at 120 m/s, are moving towards each
other on the same track. When they are 11 km apart, both drivers simultaneously apply brakes. If
the brakes decelerate each train at the rate of 3m/s2 then the distance travelled by the first train is.
(1) 1350 m (2) 2400 m (3) 4740 m (4) 8870 m
20. In the above problem, the distance travelled by the second train is
(1) 1350 m (2) 2400 m (3) 3740 m (4) 8870 m
21. In the above problem whether a collision will take place or not
(1) Collision will take place (2) There shall be no collision
(3) Collision may not take place (4) None of these
22. A body starts from rest with uniform acceleration. If its velocity after n second is v, then its
displacement in the last two seconds is
2v n 1 v n 1 v n 1 2v n 1
(1) (2) (3) (4)
n n n n

Corporate Office : B-58, Goal Building, Budha Colony, Patna-1, Help line : 9334594165/66/67 Website : www.goalinstitute.org
Kinematics [ 89 ]
23. Two particles move in a straight line towards each other with initial velocities v1 and v 2 and a1 and
a2 retardation towards each other. The maximum initial separation between the two particles so
that they may meet must be
2
v 1  v 2  v 1  v 2  v1  v 2  (v 1  v 2 )
(1) 2a1  a 2  (2) 2a1  a 2  (3) (4) 2(a1  a 2 )2
2a1 a 2
24. A point starts moving in a straight line with a certain acceleration. At a time t after beginning of
motion the acceleration suddenly becomes retardation of the same value. The time in which the
point returns to the initial point is
(1) 2t

(2) (2  2)t

t
(3)
2
(4) Cannot be predicted unless acceleration is given
25. A particle is moving in a straight line and passes through a point O with a velocity of 6ms 1. The
particle moves with a constant retardation of 2ms 2 for 4 s and there after moves with constant
velocity. How long after leaving O does the particle return to O
(1) 3s (2) 4s (3) Never (4) 8s
26. A bird flies for 4 s with a velocity of |t  2|m/s in a straight line, where t = time in seconds. It covers
a distance of
(1) 2 m (2) 4 m (3) 6 m (4) 8 m
27. The velocity of a particle is dependent on the time as v  k t 1 where k  2m/s 2. The distance
covered in first three seconds will be
(1) 18 m (2) 5 m (3) 3 m (4) 6 m
28. A particle is projected with velocity v0 along x-axis. The deceleration on the particle is proportional
to the square of the distance from the origin i.e. a   x 2. The distance at which the particle stops is
1 1
3v 0  3v o  3 3v 20  3v 20  3
(1) (2)  2  (3) (4)  2 
2   2  
29. Four marbles are dropped from the top of a tower one after the other with an interval of one second.
The first one reaches the ground after 4 seconds. When the first one reaches the ground the
distances between the first and second, the second and third and the third and forth will be
respectively
(1) 35, 25 and 15 m (2) 30, 20 and 10 m (3) 20, 10 and 5 m (4) 40, 30 and 20 m
30. A ball is dropped from the top of the tower of height h. It covers a distance of h/2 in the last second
of its motion. How long does the ball remain in air (Take g = 10 m/s2)
(1) 2s (2) (2  2)s (3) 2s (4) None of the above

Corporate Office : B-58, Goal Building, Budha Colony, Patna-1, Help line : 9334594165/66/67 Website : www.goalinstitute.org
Kinematics [ 90 ]

31. A body is dropped form height h. If t1 and t2 be the times in covering first half and next half distances
respectively, then the correct relation is

t2
(1) t1 t2 (2) t 1  2t2 (3) t1  (4) t 1  4t2
2 1
32. A balloon rises from rest with a constant acceleration g/8 . A stone is released from it when it has
risen to height h. The time taken by the stone to reach the ground is

h h 2h g
(1) 4 (2) 2 (3) (4)
g g g h
33. A ball is projected upwards from a height h above the surface of the earth with velocity v. The time
at which the ball strikes the ground is

v 2hg v 2h 
(1)  (2) 1 1 
g 2 g g

v 2gh  v 2g 
(3) 1 1 2  (4) 1 v 2  
g  v  g  h

34. Two bodies are thrown simultaneously from a tower with same initial velocity v0 , one vertically
upwards, the other vertically downwards. The distance between the two bodies after time t is
1 1
(1) 2v 0 t  gt 2 (2) 2v 0 t (3) v 0 t  gt 2 (4) v 0t
2 2
35. A body falls freely from the top of a tower. It covers 36% of the total height in the last second before
striking the ground level. The height of the tower is (take g = 10 m/s2)
(1) 50 m (2) 75 m (3) 100 m (4) 125 m
36. A particle is projected upwards. The times corresponding to height h while ascending and while
descending are t1 and t2 respectively. The velocity of projection will be

g  t1 + t 2 
(1) gt1 (2) gt2 (3) g(t1 + t2) (4)
2
37. A projectile is fired vertically upwards with an initial velocity u. After an interval of T seconds a
second projectile is fired vertically upwards, also with initial velocity u.

u u2 gT 2
(1) They meet at time t  and at a height 
g 2g 8

u T u2 gT 2
(2) They meet at time t   and at a height 
g 2 2g 8

u T u2 gT 2
(3) They meet at time t   and at a height 
g 2 2g 8
(4) They never meet

Corporate Office : B-58, Goal Building, Budha Colony, Patna-1, Help line : 9334594165/66/67 Website : www.goalinstitute.org
Kinematics [ 91 ]

Answer Exercise - 3
1. (1) 2. (1) 3. (4) 4. (1) 5. (3) 6. (4)
7. (4) 8. (1) 9. (2) 10. (4) 11. (1) 12. (2)
13. (1) 14. (4) 15. (2) 16. (3) 17. (2) 18. (1)
19. (1) 20. (2) 21. (2) 22. (4) 23. (2) 24. (2)
25. (4) 26. (2) 27. (3) 28. (4) 29. (1) 30. (2)
31. (3) 32. (2) 33. (3) 34. (2) 35. (4) 36. (4)
37. (3)

Note

Corporate Office : B-58, Goal Building, Budha Colony, Patna-1, Help line : 9334594165/66/67 Website : www.goalinstitute.org
Kinematics [ 92 ]

Exercise-4
Motion in Two Dimensions ‘O’ Level
1. A stone is projected with an initial velocity at an angle to the horizontal. A small piece separates
from the stone before the stone reaches its maximum height. Then this piece will
(1) Fall to the ground vertically
(2) Fly side by side with the parent stone along a parabolic path
(3) Fly horizontally initially and will trace a different parabolic path
(4) Lag behind the parent stone increasing the distance from it

2 3
2. The equation of motion of a projectile is y  12 x  x . . Given that g =10 ms–2, what is the range of
4
the projectile
(1) 12.4 m (2) 16 m (3) 30.6 m (4) 36.0 m
3. A body of mass m is thrown upwards at an angle  with the horizontal with velocity v. While rising
up the velocity of the mass after t seconds will be

(1) (v cos  )2  (v sin  )2 (2) (v cos   v sin ) 2  gt

(3) v 2  g 2 t 2  (2v sin )gt (4) v 2  g 2 t 2  (2v cos  )gt

4. When a particle is thrown horizontallywith velocity, u the resultant velocity of the projectile at any
time t is given by

1 2
(1) gt (2) gt (3) u2  g2 t 2 (4) u2  g2 t 2
2
5. An aeroplane is moving with a horizontal velocity u at a height h above the ground. If a packet is
dropped from it the speed of the packet when it reaches the ground will be
(1) (u2  2gh)1/2 (2) (2gh)1/2 (3) (u2 – 2gh)1/2 (4) 2gh
6. If t1 be the time taken by a body to reach maximum height clear the top of a building and t2 be the
time spent in air, then t2 : t1 will be
(1) 1:2 (2) 2:1 (3) 1 : 1 (4) 1 : 4
7. A bomb is fired from a cannon with a velocity of 1000 m/s making an angle of 30o with the horizontal.
What is the time taken by the bomb to reach the highest point
(1) 11 sec (2) 23 sec (3) 38 sec (4) 51 sec
8. A hiker stands on the edge of a cliff 490 m above the ground and throws a stone horizontally with
a speed of 15 ms–1. The time taken by the stone to reach the ground is
(1) 10 s (2) 5s (3) 12 s (4) 15 s
9. Galileo’s experiment showed that if two bodies of unequal masses are dropped from the same
height, the time required by them to reach the ground are equal. But if they are thrown vertically
upwards with the same initial velocity, the ratio of the time required to reach the ground is equal to
(1) The ratio of their masses (2) The inverse of the ratio of their masses
(3) One (4) The product of their masses

Corporate Office : B-58, Goal Building, Budha Colony, Patna-1, Help line : 9334594165/66/67 Website : www.goalinstitute.org
Kinematics [ 93 ]
10. A projectile fired with initial velocity u at some angle  , has a range R. If the initial velocity be
doubled at the same angle of projection, then the range will be
(1) 2R (2) R/2 (3) R (4) 4R
11. Three identical balls are thrown with same speed at angles of 15 , 45 and 75o with the horizontal
o o

respectively. The ratio of their distances from the point of projection to the where they hit the
ground will be
(1) 1: 2 :1 (2) 1 : 2 : 1 (3) 2 : 4 : 3 (4) 1:2: 3
12. Five balls A, B, C, D and E are projected with the same speed making angles of 10°, 30°, 45°, 60°,
80° respectively with the horizontal. Which ball will strike the ground at the farthest point
(1) B (2) A (3) E (4) C
o
13. An object is thrown along a direction inclined at an angle of 45 with the horizontal direction. The
horizontal range of the particle is equal to
(1) Vertical height (2) Twice the vertical height
(3) Thrice the vertical height (4) Four times the vertical height
o
14. A projectile is thrown at an angle of 40 with the horizontal and its range is R1. Another projectile is
thrown at an angle 40o with the vertical and its range is R2. What is the relation between R1 and R2
(1) R1 = R2 (2) R1 = 2 R2 (3) R2 = 2 R1 (4) R1 = 4 R2 /5
15. Two particles are projected from the same point with the same speed at different angle 1 and 2
to the horizontal. They have the same horizontal range. Their times of flight are t1 and t2 respectively.
Then
t1 ta n  1 1
t 1
t
(1) 1 + 2 = 90o (2) t2

ta n  2 (3) t  tan 2 (4) t  ta n  1
2 2

16. A projectile thrown from a height of 10 m with velocity of 2m/s , the projectile will fall, from the foot
of projection, at distance (g = 10 m/s2)
(1) 1 m (2) 2 m (3) 3 m (4) 4 m
17. On throwing a ball from the top of a vertical tower with a horizontal velocity of 20 m/s, it hits the
earth at a distance of 50 m from the foot of the tower. If the ball is thrown with the same horizontal
velocity from the top of another tower of height four times the height of first tower, the ball will hit the
earth at a distance of
(1) 50 m (2) 100 m (3) 200 m (4) 150 m
o
18. A cricketer hits a ball with a velocity 25 m/s at 60 above the horizontal. How far above the ground
it passes over a fielder 50 m from the bat (assume the ball is struck very close to the ground)
(1) 8.2 m (2) 9.0 m (3) 11.6 m (4) 12.7 m
19. Which of the following is largest, when the height attained by the projectile is the largest
(1) Range (2) Time of flight
(3) Angle of projectile with vertical (4) None of these
20. In broad jumping does it matter how high you jump? What factors determine the span of the jump
(1) u and  (2) u and h (3) No, h and  (4) None of these
21. A bullet of mass m is fired from a point at an angle of 45o with the horizontal with a velocity v. When
it reaches the ground, the change in its vertical component of momentum is
1
(1) Zero (2) mv (3) mv 2 (4) 2 mv
2

Corporate Office : B-58, Goal Building, Budha Colony, Patna-1, Help line : 9334594165/66/67 Website : www.goalinstitute.org
Kinematics [ 94 ]

22. A 150 m long train is moving to north at a speed of 10 m/s. A parrot flying towards south with a
speed of 5 m/s crosses the train. The time taken by the parrot the cross to train would be:
(1) 30 s (2) 15 s (3) 8 s (4) 10 s
23. A swimmer can swim in still water with speed u and the river is flowing with velocity v/2. To cross
the river in shortest time, he should swim making angle  with the upstream. What is the ratio of
the time taken to swim across the shortest time to that is swimming across over shortest distance
(1) cos  (2) sin  (3) tan  (4) cot 
24. The speed of a boat is 5 km/h in still water. It crosses a river of width 1 km along the shortest
possible path in 15 minutes. The velocity of the river water is
(1) 1 km/h (2) 3 km/h (3) 4 km/h (4) 5 km/h
25. A river is flowing from east to west at a speed of 5 m/min. A man on south bank of river, capable
of swimming 10m/min in still water, wants to swim across the river in shortest time. He should
swim (1) Due north (2) Due north-east
(3) Due north-east with double the speed of river (4) None of these
26. A person to reach the exactly opposite point on the bank of a stream is swimming with a speed of
0.5 m/s at an angle of 1200 with the direction of flow of water. The speed of water in the stream is
(1) 1 m/s (2) 0.5 m/s (3) 0.25 m/s (4) 0.433 m/s
27. A moves with 65 km/h while B is coming back of A with 80 km/h. The relative velocity of B with
respect to A is
(1) 80 km/h (2) 60 km/h (3) 15 km/h (4) 145 km/h
28. A man crosses a 320 m wide river perpendicular to the current in 4 minutes. If in still water he can
swim with a speed 5/3 times that of the current, then the speed of the current, in m/min is
(1) 30 (2) 40 (3) 50 (4) 60.
29. A thief is running away on a straight road on a jeep moving with a speed of 9 m/s. A police man
chases him on a motor cycle moving at a speed of 10 m/s. If the instantaneous separation of jeep
from the motor cycle is 100 m, how long will it take for the policemen to catch the thief
(1) 1 second (2) 19 second (3) 90 second (4) 100 second
30. A man can swim with velocity v relative to water. He has to cross a river of width d flowing with a
velocity u (u > v). The distance through which he is carried down stream by the river is x. Which of
the following statement is correct
du
(1) If he crosses the river in minimum time x 
v
du
(2) x can not be less than
v
 v 
(3) For x to be minimum he has to swim in a direction making an angle of  sin1   with the
2 u
direction of the flow of water
 v 
(4) x will be max. if he swims in a direction making an angle of  sin1   with direction of the
2 u
flow of water

Corporate Office : B-58, Goal Building, Budha Colony, Patna-1, Help line : 9334594165/66/67 Website : www.goalinstitute.org
Kinematics [ 95 ]

Answer Exercise-4
1. (2) 2. (2) 3. (3) 4. (3) 5. (1) 6. (2)
7. (4) 8. (1) 9. (3) 10. (4) 11. (2) 12. (4)
13. (4) 14. (1) 15. (4) 16. (2) 17. (2) 18. (1)
19. (2) 20. (1) 21. (3) 22. (4) 23. (2) 24. (2)
25. (1) 26. (3) 27. (3) 28. (4) 29. (4)
30. (1) (3)

Note

Corporate Office : B-58, Goal Building, Budha Colony, Patna-1, Help line : 9334594165/66/67 Website : www.goalinstitute.org
Kinematics [ 96 ]

Exercise – 5
Motion in Two Dimensions ‘A’ Level
   
1. A particle is moving in a plane with velocity given by u  u0 i  (a cos t) j where i , j are unit
vectors along x and y axes respectively. The trajectory of the particle if the particle starts from
origin at t = 0 will be

x  x 
(1) y  asin  (2) y  acos  u 
 u0   0
(3) y = tanx (4) y = costx
2. An object is projected with a velocity of 20 m/s making an angle of 45o with horizontal. The equation
for the trajectory is h = Ax – Bx2 where h is height, x is horizontal distance, A and B are constants.
The ratio A : B is (g = 10 m/s2)
(1) 1 : 5 (2) 5 : 1
(3) 1 : 40 (4) 40 : 1
3. A ball is dropped from the top of a tower in a high speed wind. The wind exerts a steady force on
the ball. The path followed by the ball will be
(1) Parabola (2) Circular arc
(3) Elliptical arc (4) Straight line
4. A particle is projected with a speed V from a point O making an angle of 30o with the vertical. At the
same instant, a second particle is thrown vertically upwards from a point A. The two particle reach
H, the highest point on the parabolic path of particle simultaneously.

V V
Then ratio is H
v
o
(1) (2) 30
3 2 23
60o v
2 3
(3) (4) O A
3 2
5. A particle is thrown upward with a speed u at an angle  with the horizontal. When the particle
makes an angle  with the horizontal, its speed changes to v, then
(1) v = u cos cos (2) v = u cossec
(3) v = u cos (4) v = u seccos
6. Mr. Naveen kicked off a football with an initial speed 19.6 m/s at a projection angle 45o. A receiver
on the goal line 67.4 m away in the direction of the kick starts running to meet the ball at that
instant. What must be his speed so that he could catch the ball before hitting the ground

(1) 2.82 m/s (2) 2 / 2m/ s (3) 39.2 m/s (4) 10 m/s
7. Two balls of same mass are thrown horizontally from the top of a tower in the opposite direction
with velocities 3 m/s and 4 m/s. The distance between the balls, when their velocities are mutually
perpendicular will be nearest to
(1) 10 m (2) 7 m
(3) 5m (4) 2.5 m

Corporate Office : B-58, Goal Building, Budha Colony, Patna-1, Help line : 9334594165/66/67 Website : www.goalinstitute.org
Kinematics [ 97 ]
8. A motorcyclist starts from the bottom of a slope of angle 45o
to cross the valley PR as shown in the
figure. The width of the valley is 90 m and length of the slope is 80 2m . The minimum velocity at
point O required to clear the valley will be
Q
P R

45o
O

(1) 70m /s (2) 30m /s (3) 50m/s (4) 100m/s


9. From the top of a tower of height h a body of a mass m is projected in the horizontal direction with
a velocity v. It falls on the ground at a distance x from the tower. If a body of mass 2 m is projected
from the top of another tower of height 2 h in the horizontal direction so that it falls on the ground at
a distance 2x from the tower, the horizontal velocity of the second body is

v v
(1) 2v (2) 2v (3) (4)
2 2

10. A particle is projected with a speed 2 gh so that it clears two walls of equal height h which are at
a distance 2h from each other. The time taken by the particle to pass between the two walls is

2h 2h h h
(1) (2) (3) 2 (4) 2 g
g g g
11. A projectile is thrown in the upward direction making an angle of 60o with the horizontal direction
with a velocity of 147 ms–1. Then the time after which its inclination with the horizontal is 45o is
(1) 15 s (2) 10.98 s (3) 5.49 s (4) 2.745 s
12. A cannon ball is fired with a velocity v in a direction making an angle  with the horizontal. At the
highest point of its path it breaks into two parts of equal masses. One of the parts retraces the initial
path of the ball. The speed of the second part immediately after explosion in m/s will be

3 3
(1) v cos  (2) v cos  (3) 2v cos  (4) 3v cos 
2 2
13. From the top of a tower of height 40 m a ball is projected upwards with a speed of 20 m/s at an
angle of elevation of 30o. Then the ratio of the total time taken by the ball to hit the ground to its time
of flight (time taken to come back to the same elevation) is (take g = 10 ms2)
(1) 2 : 1 (2) 3 : 1 (3) 3 : 2 (4) 4 : 1
14. A shell is fired vertically upwards with a velocity v1 from the deck of a ship travelling at a speed of
v 2. A person on the shore observes the motion of the shell as parabola whose horizontal range is
given by
2v21 v 2 2v1 v22 2v1 v2 2v21 v 2
(1) g (2) g (3) g (4) g
15. A boy standing on a long rail road car throws a ball straight upward. The car is moving on the
horizontal road with an acceleration of 1 m/s2 and the projection velocity in the vertical direction is
9.8 m/s. How far behind the boy will the ball fall on the car
(1) 1 m (2) 2 m (3) 4 m (4) 0.5 m
Corporate Office : B-58, Goal Building, Budha Colony, Patna-1, Help line : 9334594165/66/67 Website : www.goalinstitute.org
Kinematics [ 98 ]
16. A projectile has a maximum range of 16 km. At the highest point of its motion, it explodes into two
equal masses. One mass drops vertically downwards the horizontal distance covered by the other
mass from the time of explosion is
(1) 8 km (2) 16 km (3) 24 km (4) 32 km
v
17. Two projectiles A and B thrown with velocities v and have the same range. If B is thrown at an
2
angle of 15o to the horizontal, A must have been thrown at an angle

1 1  1  1 1 1  1
(1) sin1   (2) sin1   (3) 2sin  4 (4) 2 sin  8
16  4    
18. A body is projected with a speed u in such a direction that the maximum height obtained is equal to
its horizontal range. The horizontal range is
8u2 5u2 3u2 u2
(1) 17g (2) 17g (3) 17g (4) 2g

19. A body of mass m thrown horizontally with velocity v, from the top of tower of height h touches the
level ground at a distance of 250 m from the foot of the tower. A body of mass 2m, thrown horizontally
with a velocity v/2, from the top of tower of height 4 h will touch the level ground at a distance from
the foot of tower
(1) 250 m (2) 500 m (3) 125 m (4) m
20. A small block of mass 0.1 kg is pressed against a horizontal spring fixed at one end to compress
the spring through 5.0 cm as shown in figure. The spring constant is 100 N/m. When released the
block moves horizontally till it leaves the spring. Where will it hit the ground 2 m below the spring
(1) Vertically below the edge on which the mass is resting
(2) At a horizontal distance of 1.0 m from free end of the spring
(3) At a horizontal distance of 2 m from free end of the spring 2m

(4) At a horizontal distance of 0.5 m from free end of the spring


21. From a point on the ground at a distance 2 metres from the foot of a vertical wall, a ball is thrown at
an angle of 45o which just clears the top of the wall and afterward strikes the ground at a distance
4m on the other side. The height of the wall is
2 3 1 4
(1) m (2) m (3) m (4) m
3 4 3 3
22. A ball is thrown from the top of a tower with an initial velocity of 10 m/s at an angle of 30o above the
horizontal. It hits the ground at a distance of 17.3 m from the base of the tower. The height of the
tower (g = 10 m/s2) will be
(1) 10 m (2) 12 m
(3) 110 m (4) 100 m
23. A ball is projected from the ground at a speed of 10 m/s making angle 30o with the horizontal.
Another ball is simultaneously released from a point on the vertical line along the maximum height
of the projectile. Both the balls collide at the maximum height of the projectile. What was the initial
height of the second ball (g = 10 m/s2)
(1) 1.0 m (2) 1.25 m
(3) 2.0 m (4) 2.5 m
Corporate Office : B-58, Goal Building, Budha Colony, Patna-1, Help line : 9334594165/66/67 Website : www.goalinstitute.org
Kinematics [ 99 ]
24. A hose shoots a stream of water at an angle of 60o the horizontal with a velocity of 20 m/s. Water
will strike a wall at a distance of 10 m at a height
(1) 5.36 m (2) 10.22 m (3) 12.42 m (4) 16.84 m
25. A particle having a mass of 0.5 kg is projected under gravity with a speed of 98 m/sec at an angle
of 60o. The magnitude of the change in momentum in N-sec of the particle after 10 seconds is
(1) 0.5 (2) 49 (3) 98 (4) 490
26. A projectile is fired at 30o with momentum p, neglecting friction the change in momentum on return
to the ground will be
(1) Zero (2) 30% (3) 60% (4) 100%
27. A ball is thrown up with a given angle to the horizontal. Then the total change of momentum by the
instant it returns to the ground is equal to
(1) Acceleration due to gravity × total time of flight
(2) Weight of the ball × half the time of flight
(3) Weight of the ball × total time of flight
(4) None of these
28. A racing car is travelling along a track at a constant speed of 40/ms. A T.V. camera man is recording
the event from a distance of 30 m directly away from the track as shown in figure. In order to keep
the car under view in the position shown, the angular speed with which the camera should be
rotated, is
Car
(1) 4/3 rad / sec 40 m/s

(2) 3/4 rad / sec


30m
30o
(3) 8/ 3 3 rad/ sec
T.V. Camera
(4) 1 rad/sec
29. An aeroplane is moving in a horizontal circle with a uniform speed of 141 km/hr. Change in its
velocity in one fourth revolution, from its initial direction will be
(1) 100 km/hr at an angle of 90o (2) 141 km/hr at an angle of 135o
(3) 200 km/hr at an angle of 135o (4) Zero
30. The driver of a car travelling at velocity v suddenly sees a broad wall in front of him at a distance d.
He should
(1) Brake sharply (2) Turn sharply (3) (1) and (2) both (4) None of the above

Answer Exercise-5
1. (1) 2. (4) 3. (4) 4. (3) 5. (2) 6. (4)
7. (4) 8. (3) 9. (2) 10. (4) 11. (3) 12. (4)
13. (1) 14. (3) 15. (2) 16. (2) 17. (4) 18. (1)
19. (1) 20. (2) 21. (4) 22. (1) 23. (4) 24. (3)
25. (2) 26. (4) 27. (3) 28. (4) 29. (3) 30. (1)

Corporate Office : B-58, Goal Building, Budha Colony, Patna-1, Help line : 9334594165/66/67 Website : www.goalinstitute.org
Kinematics [ 100 ]

Exercise-6
Competitive Corner
1. The position x of a particle with respect to time along x–axis is given by x = 9t2 – t3 where x is in
metres and t in seconds. What will be the position of this particle when it achieves maximum
speed along the +x direction ? [AIPMT 2007]
(1) 54 m (2) 81 m (3) 24 m (4) 32 m
2. A car moves from X to Y with a uniform speed vu and returns to Y with a uniform speed vd. The
average speed for this round trip is [AIPMT 2007]

v dv u vu  vd 2v d vu
(1) vu v d (2) vd  vu (3) (4) v  v
2 d u

 t
3. A particle moving along x–axis has acceleration f, at time t, given by f = f0 1  T  , where f0 and T are
 
constants. The particle at t = 0 has zero velocity. In the time interval between t = 0 and the instant when
f = 0, the particle’s velocity (vx) is [AIPMT 2007]

1 1
(1) f T2 (2) f0 T2 (3) f T (4) f0 T
2 0 2 0
4. A particle starting from the origin (0, 0) moves in a straight line in (x, y) plane. Its coordinates at a
later time are ( 3, 3) The path of the particle makes with the x-axis angle of [AIPMT 2007]
(1) 45° (2) 60° (3) 0° (4) 30°

4
5. The distance travelled by a particle starting from rest and moving with an acceleration ms –2, in
3
the third second is [AIPMT 2008]
10 19
(1) m (2) m (3) 6m (4) 4 m
3 3

6. A particle moves in a straight line with a constant acceleration. It changes its velocity from 10 ms–1 to
20 ms –1 while passing through a distance 135 m in t second. The value of t is
[AIPMT 2008]
(1) 12 s (2) 9s (3) 10s (4) 1.8s
7. A particle of mass m is projected with velocity v making an angle of 45° with the horizontal. When the
particle lands on the level ground the magnitude of the change in its momentum will be [AIPMT 2008]
(1) mv 2 (2) zero (3) 2mv (4) mv/ 2
8. A bus is moving with a speed of 10 ms–1 on a straight road. A scooterist wishes to overtake the
bus in 100 s. If the bus is at a distance of 1 km from the scooterist, with what speed should the
scooterist chase the bus? AIPMT 2009]
–1 –1 –1
(1) 40 ms (2) 25 ms (3) 10 ms (4) 20 ms –1

Corporate Office : B-58, Goal Building, Budha Colony, Patna-1, Help line : 9334594165/66/67 Website : www.goalinstitute.org
Kinematics [ 101 ]
9. A particle starts its motion from rest under the action of a constant force. If the distance covered in
first 10 seconds is S1 and that covered in the first 20 seconds is S2, then [AIPMT 2009]
(1) S2 = 3S1 (2) S2= 4S1 (3) S2 = S1 (4) S2 = 2S1
10. A particle moves a distance x in time t according to equation x = (t + 5)–1. The acceleration of
particle is proportional to [AIPMT 2010]
3/2 2 -2
(1) (velocity) (2) (distance) (3) (distance) (4) (velocity)2/3
11. A ball is dropped from a high rise platform at t = 0 starting from rest. After 6 seconds another ball
is thrown downwards from the same platform with a speed v. The two balls meet at t = 18 s. What
is the value of v ? (Take g = 10 m/s2) [AIPMT 2010]
(1) 75 m/s (2) 55 m/s (3) 40 m/s (4) 60 m/s
  ^ ^
12. A particle has initial velocity (3 i  4 j ) and has acceleration (0.4 i + 0.3 j ), Its speed after 10 s is
[AIPMT 2010]
(1) 7 units (2) 7 2 units (3) 8.5 units (4) 10 units

13. Six vectors, a through f have the magnitudes and directions indicated in the figure. Which of the
following statements is true? [AIPMT 2010]
  
(1) b c  f

    b 
(2) d c  f a c

   f
(3) de  f  
d e
  
(4) be  f
14. The speed of a projectile at its maximum height is half of its initial speed. The angle of projection is
[AIPMT 2010]
(1) 60° (2) 15° (3) 30° (4) 45°
15. A particle moves in x–y plane according to rule x = a sint and y = a cost. The particle follows
[AIPMT 2010]
(1) an elliptical path.
(2) a circular path.
(3) a parabolic path.
(4) a straight line path inclined equally to x and y–axis.
16. A boy standing at the top of a tower of 20 m height drops a stone. Assuming g = 10 ms–2, the
velocity with which it hits the ground is [AIPMT 2011]
(1) 10.0 m/s (2) 20.0 m/s (3) 40.0 m/s (4) 5.0 m/s
17. A particle moves in a circle of radius 5 cm with constant speed and time period 0.2s. The acceleration
of the particle is [AIPMT 2011]
(1) 15 m/s 2 (2) 25 m/s 2 (3) 36 m/s2 (4) 5 m/s 2
18. A missile is fired for maximum range with an initial velocity of 20 m/s. If g = 10 m/s2, the range of
the missile is [AIPMT 2011]
(1) 40 m (2) 50 m (3) 60m (4) 20m
19. A body is moving with velocity 30 m/s towards east. After 10 seconds its velocity becomes
40 m/s towards north. The average acceleration of the body is [AIPMT 2011]
2 2 2
(1) 1 m/s (2) 7 m/s (3) 7 m/s (4) 5m/s 2

Corporate Office : B-58, Goal Building, Budha Colony, Patna-1, Help line : 9334594165/66/67 Website : www.goalinstitute.org
Kinematics [ 102 ]
20. A projectile is fired at an angle of 45° with the horizontal. Elevation angle of the projectile at its
highest point as seen from the point of projection [AIPMT 2011]
 1  3
(1) 45° (2) 60° (3) tan–1   (4) tan–1  2 
 2  
21. The motion of a particle along a straight line is described by equation x = 8 + 12t – t3 where x is in
metre and t in second. The retardation of the particle when its velocity becomes zero is
[AIPMT 2012]
(1) 24 ms –2 (2) zero (3) 6 ms –2 (4) 12 ms –2
22. A particle shows distance - time curve as given in this figure. The maximum instantaneous velocity of
the particle is around the point [AIPMT 2012]

D
Distance

S C

A B
t Time
(1) D (2) A (3) B (4) C
23. A stone falls freely under gravity. It covers distances h1, h2 and h3 in the first 5 seconds, the next
5 seconds and the next 5 seconds respectively. The relation between h1, h2 and h3 is :
[NEET-2013]
(1) h1 = h2 = h3 (2) h1 = 2h2 = 3h3
h2 h3
(3) h1   (4) h2 = 3h1 and h3 = 3h2
3 5
 
24. The velocity of a projectile at the initial point A(2 i  j)m / s . Its velocity (in m/s) at point B is :
[NEET-2013]

B
X
A

       
(1) 2 i 3 j (2) 2 i  3 j (3) 2 i  3 j (4) 2 i  3 j
25. The horizontal range and the maximum height of a projectile are equal. The angle of projection of
the projectile is
 1
(1)  = tan–1   (2)  = tan–1 (4) (3)  = tan–1 (2) (4)  = 450
 4

Corporate Office : B-58, Goal Building, Budha Colony, Patna-1, Help line : 9334594165/66/67 Website : www.goalinstitute.org
Kinematics [ 103 ]

^ ^ ^ ^
26. A particle has initial velocity (2 i + 3 j ) and acceleration (0.3 i + 0.2 j ). The magnitude of velocity
after 10 seconds will be
(1) 9 2 units (2) 5 2 units (3) 5 units (4) 9 units
27 From a tower of height H, a particle is thrown vertically upwards with a speed u. The time taken by
the particle, to hit the ground, is n times that taken by it to reach the highest point of its path. The
relation between H, u and n is : [AIPMT 2014]
(1) 2gH = n u 2 2 (2) gH = (n – 2) u2 2 (3) 2gH = nu (n – 2) (4) gH = (n – 2) u2
2

28. A projectile is fired from the surface of the earth with a velocity of 5 ms–1 and angle  with the
horizontal. Another projectile fired from another planet with a velocity of 3 ms–1 at the same
angle follows at trajectory which is identical with the trajectory of the projectile fired from the
earth. The value of the accel eration due to gra vity on the planet is (in ms –2 ) i s :
(given g = 9.8 ms –2) [AIPMT-2014]
(1) 3.5 (2) 5.9
(3) 16.3 (4) 110.8
29. A particle of unit mass undergoes one dimensional motion such that its velocity varies according to
v  x    x 2 n where  and n are constants and x is the position of the particle. The aceleration of
the particle as a function of x, is given by : [NEET-2015]
(1) 2n 2 x 4n1 (2) 22 x 2n1 (3) 2n 2e 4n1 (4) 2n 2 x 2n1
30. A ship A is moving Westwards with a speed of 10 km/h and a ship B 100 km South of A, is moving
Northwards with a speed of 10 km/h. The time after which the distance between them becomes
shortest, is : [NEET-2015]
(1) 5h (2) 5 2h (3) 10 2 h (4) 0 h

31. The position vector of a particle R as a function of time is given by :
  
R  4 sin  2 t  i  4 cos  2t  j
 
Where R is in metres, t is in seconds and i and j denote unit vectors along x and y-direction
respectively. Which one of the following statements is wrong for the motion of particle?
[NEET-2015]
(1) Path of the particle is a circle of radius 4 metre

(2) Acceleration vectors is along  R

v2
(3) Magnitude of acceleration vector is where v is the velocity of particle
R
(4) Magnitude of the velocity of particle is 8 metre/second
32. A bullet of mass 10 g moving horizontally with a velocity of 400 ms–1 strikes a wooden block of
mass 2 kg which is suspended by a light inextensible string of length 5 m. As a result, the center of
gravity of the block is found to rise a vertical distance of 10 cm. The speed of the bullet after it
emerges out horizontally from the block will be : [NEET-2016-II]
(1) 120 ms –1 (2) 160 ms –1 (3) 100 ms –1 (4) 80 ms –1
Corporate Office : B-58, Goal Building, Budha Colony, Patna-1, Help line : 9334594165/66/67 Website : www.goalinstitute.org
Kinematics [ 104 ]
33. Two cars P and Q start from a point at the same time in a straight line and their positions are
represented by Xp(t) = at + bt2 and XQ(t) = ft – t2. At what time do the ears have the same velocity?
[NEET-2016-II]

a f f a af a f
(1) 2 1  b  (2) 2 1  b  (3) (4) 2 b  1
1 b  
34. If the velocity of a particle is v = At + Bt2, where A and B are constants, then the distance travelled
by it between 1 s and 2 s is : [NEET-2016-I]
3 3 7 A B
(1) A  4B (2) 3A + 7B (3) A B (4) 
2 2 3 2 3
  
35. A particle moves so that its position vector is given by r  cos  t x  sin t y where  is a constant
which of the following is true? [NEET-2016-I]

(1) Velocity and acceleration both are perpendicular to r

(2) Velocity and acceleration both are parallel to r

(3) Velocity is perpendicular to r and acceleration is directed towards the origin

(4) Velocity is perpendicular to r and acceleration is directed away from the origin
36. A uniform circular disc of radius 50 cm at rest is free to turn about an axis which is perpendicular to
its plane and passes through its center. It is subjected to a torque which produces a constant
angular acceleration of 2.0 rad s–2. Its net acceleration in ms–2 at the end of 2.0 s is approximately:
[NEET-2016-I]
(1) 8.0 (2) 7.0 (3) 6.0 (4) 3.0
37. Preeti reached the metro station and found that the escalator was not working. She walked up the
stationary escalator in time t1. On other days, if she remains stationary on the moving escalator,
then the escalator takes her up in time t2. The time taken by her to walk up on the moving escalator
will be : (NEET-2017)

t1  t 2 t1t 2 t 1t 2
(1) t1 – t2 (2) (3) t2  t1 (4) t  t
2 2 1

38. The x and y coordinates of the particle at any time are x = 5t – 2t2 and y = 10t respectively, where
x and y are in meters and t in seconds. The acceleration of the particle at t = 2s is :(NEET-2017)
(1) –8 m/s 2 (2) 0 (3) 5 m/s 2 (4) –4 m/s 2
39. A beam of light from a source L is incident normal on a plane mirror fixed at a certain distance x
from the source. The beam is reflected back as a spot on scale placed just above the source L.
When the mirror is rotated through a smal angle , the spot of the light is found to move through a
distance y on the scale. The angle is given by : (NEET-2017)

x y y x
(1) y
(2) (3) (4) 2y
2x x

Corporate Office : B-58, Goal Building, Budha Colony, Patna-1, Help line : 9334594165/66/67 Website : www.goalinstitute.org
Kinematics [ 105 ]
40. Which of the following velocity-time graphs shows a realistic situation for a body in motion?
[AIIMS 2007]

v v v

(1) (2) (3) (4)


t t t

41. The displacement time graph of a particle moving along a straight line is drawn below. The
accelerations of the particle during the regions OP, PQ and QR are [AIIMS 2008]

R
OP PQ QR

Displacement
(1) – 0 +
P Q
(2) + 0 +
(3) + 0 –
O Time
(4) – 0 –
42. A particle moves along with x-axis. The position x of article with respect to time t from origin given
by x = b0 + b1t + b2t2. The acceleration of particle is : (AIIMS -2017)
(1) b0 (2) b1 (3) b2 (4) 2b 2

 
43. A projectile is given an initial velocity of i  2 j . The cartesian equation of its path is
(Take g = 10 s–2): (AIIMS-2017)
(1) y = x – 5x 2 (2) y = 2x – 5x2 (3) y = 2x – 15x2 (4) y = 2x – 25x2
44. The velocity of a particle is v = v0 + gt + ft2. If its position is x = 0 at t = 0, t = 0, then its displacement
after unit time (t = 1) is [AIEEE 2007]
g f g
(1) v 0  2g  3f v0  
(2) (3) v 0 + g + f (4) v 0   f
2 3 2
45. A body is at rest at x = 0. At t = 0, it starts moving in the positive x-direction with a constant
acceleration. At the same instant another body passes through x = 0 moving in the positive
x-direction with a constant speed. The position of the first body is given by x1(t) after time t and that
of the second body by x2(t) after the same time interval. Which of the following graphs correctly
describes (x 1 – x 2) as a function of time ? [AIEEE 2008]
(x1 –x2 ) (x1 –x2 ) (x1 –x2 ) (x1 –x2 )

(1) (2) (3) (4)


t t t t
O O O O
46. Consider a rubber ball freely falling from a height h = 4.9 m onto a horizontal elastic plate. Assume
that the duration of collision is negligible and the collision with the plate is totally elastic. Then the
velocity as a function of time the height as function of time will be [AIEEE 2009]

Corporate Office : B-58, Goal Building, Budha Colony, Patna-1, Help line : 9334594165/66/67 Website : www.goalinstitute.org
Kinematics [ 106 ]

v
v1 

(1)
O t 

v
+v1 y

O t
(2)
–v1 t

V
+V1 y

t1 2t1 3t1 4t1 5t1 6t1


(3) h

–V1 t
v y
+v1

O t t t
(4) 1 2t1 4t1
–v1

dv
47. An object, moving with a speed of 6.25 m/s, is decelerated at a rate given by  –2.5 v where v
dt
is the instantaneous speed. The time taken by the object, to come to rest, would be
[AIEEE 2011]
(1) 2 s (2) 4 s
(3) 8 s (4) 1 s
48. A particle of mass m is projected with a velocity v making an angle of 30° with the horizontal. The
magnitude of angular momentum of the projectile about the point of projection when the particle is
at its maximum height h is [AIEEE ]

3 mv 2
(1) (2) zero
2 g

mv 3 3 mv 3
(3) (4)
2g 16 g
49. A water fountain on the ground sprinkles water all around it. If the speed of water coming out of the
fountain is v, the total maximum area around the fountain that gets wet is [AIEEE]
v4  v4 v2 v2
(1)  (2) (3)  g 2 (4)  g
g2 2 g2

Corporate Office : B-58, Goal Building, Budha Colony, Patna-1, Help line : 9334594165/66/67 Website : www.goalinstitute.org
Kinematics [ 107 ]


 ^ ^
50. A particle is moving with velocity v  k y i  x j , where k is a constant. The general equation for its
 
path is [AIEEE]
2 2
(1) y = x + constant (2) y = x + constant
(3) xy = constant (4) y 2 = x 2 + constant
51. A small particle of mass m is projected at an angle  with the x-axis with an initial velocity v0 in the
v 0 sin
x-y plane as shown in the figure. At a time t  , the angular momentum of the particle is
g
[AIEEE]

v0


x

^ ^
(1) –mgv 0 t2 cos  j (2) –mgv 0 tcos k

^ ^
1 1
(3) – mgv 0 t2 cos k (4) mgv 0 t 2 cos  i
2 2
52. A point P moves in counter-clockwise direction on a circular path as shown in the figure. The
movement of P is such that it sweeps out a length s = t3 + 5, where s is in metre and t is in second.
The radius of the path is 20 m. The acceleration of P when t = 2s is nearly
[AIEEE]
y
B
P(x, y)
m
20

x
O A

(1) 13 ms –2 (2) 12 ms –2 (3) 7.2 ms –2 (4) 14 ms –2


53. A particle is projected at 60° to the horizontal with a kinetic energy K. the kinetic energy at the
highest point is

K K
(1) K (2) zero (3) (4)
4 2
54. A particle is moving such that its position coordinates (x, y) are (2m, 3m) at time t = 0, (6m, 7m) at

time t = 2s and (13m, 14m) at time t = 5s (13m, 14m) at time t = 5 s. Average velocity vector Vav  
from t = 0 to t = 5 s is : [JEE-MAIN-2014]

Corporate Office : B-58, Goal Building, Budha Colony, Patna-1, Help line : 9334594165/66/67 Website : www.goalinstitute.org
Kinematics [ 108 ]

1   
 7  
  11  
 
(1)
5
13 i 14 j (2)
3
ij  
(3) 2 i  j (4)
5
ij

55. Two stones are thrown up simultaneously from the edge of a cliff 240 m high with initial speed of
10 m/s and 40 m/s respectively. Which of the following graph best represents the time variation of
relative position of the second stone with respect to the first ? (assume stones do not rebound
after hitting the ground and neglect air resistance, take g = 10 m/s2) (The figures are schematic
and not drawn to scale.) (JEE-Main-2015)

(1) (2)

(3) (4)

56. Two cars P and Q start from a point at the same time in a straight line and their positions are
represented by XP(t) = at + bt2 and XQ(t) = ft – t2. At what do the cars have the same velocity?
(JEE-Main-2016)

af a f a f f a
(1) (2) 2  b  1 (3) 2 1  b (4) 2 1  b
1 b    
57. If the velocity of a particle is   At  Bt 2 , where A and B are the constants, then the distance
travelled by it between 1s and 2s is : (JEE-Main-2016)
3 3 7 A B
(1) A  4B (2) 3A + 7B (3) A B (4) 
2 2 3 2 3
58. A body is thrown vertically upwards. Which one of the following graphs correctly represent the
velocity vs time? (JEE-Main-2017)

v v

t t
(1) (2)

Corporate Office : B-58, Goal Building, Budha Colony, Patna-1, Help line : 9334594165/66/67 Website : www.goalinstitute.org
Kinematics [ 109 ]

v v

t t
(3) (4)

59. A body is projected at t = 0 with a velocity 10 ms–1 at an angle of 60° with the horizontal. The radius
of curvature of its trajectory at t = 1 s is R. Neglecting air resistance and taking acceleration due to
gravity g = 10 ms–2, the value of R is [JEE-Main- 2019]
(1) 5.1 m (2) 2.5 m (3) 2.8 m (4) 10.3 m
 
   
60. A particle moves from the point  2.0 i  4.0 j  m , at t = 0, with an initial velocity  5.0 i  4.0 j  ms 1 .
   
 
  2
It is acted upon by a constant force which produces a constant acceleration  4.0 i  4.0 j  ms .
 
What is the distance of the particle from the origin at time 2 s? [JEE-Main- 2019]
(1) 20 2 m (2) 15 m (3) 10 2 m (4) 5 m
61. Two guns A and B can fire bullets at speeds 1 km/s and 2 km/s respectively. From a point on a
horizontal ground, they are fired in all possible directions. The ratio of maximum areas covered by
the bullets fired by the two guns, on the ground is [JEE-Main- 2019]
(1) 1 : 4 (2) 1 : 8 (3) 1 : 2 (4) 1 : 16
62. A particle starts from the origin at time t = 0 and moves along the positive x-axis. The graph of
velocity with respect to time is shown in figure. What is the position of the particle time t = 5 s?
[JEE-Main- 2019]

v
(m/s)

3
2
1

0 1 2 3 4 5 6 7 8 9 10 t(s)

(1) 9 m (2) 6 m (3) 10 m (4) 3 m


63. In a car race on straight road, car A takes a time t less than car B at the finish and passes finishing
point with a speed ‘v’ more than that of car B. Both the cars start from rest and travel with constant
acceleration a1 and a2 respectively. Then ‘v’ is equal to: [JEE-Main- 2019]

a1  a2 2a1a 2
(1) t (2) t (3) 2a 1a2 t (4) a1a 2 t
2 a1  a2

Corporate Office : B-58, Goal Building, Budha Colony, Patna-1, Help line : 9334594165/66/67 Website : www.goalinstitute.org
Kinematics [ 110 ]
64. The stream of a river is flowing with a speed of 2 km/h. A swimmer can swim at a speed of
4 km/h. What should be the direction of the swimmer with respect to the flow of the river to cross
the river straight? [JEE-Main- 2019]
(1) 60° (2) 90° (3) 150° (4) 120°
65. A ball is thrown vertically up (taken as + z-axis) from the ground. The correct momentum-height
(p-h) diagram is: [JEE-Main- 2019]

P P P P

h h h h
(1) O (2) O (3) O (4) O

66. A shell is fired from a fixed artillery gun with an initial speed u such that it hits the target on the
ground at a distance R from it. If t1 and t2 are the values of the time taken by it to hit the target in
two possible ways, the product t1t2 is : [JEE-Main- 2019]
R 2R R R
(1) (2) (3) (4)
2g g g 4g
67. The trajectory of a projectile near the surface of the earth is given as y = 2x – 9x2. If it were
launched at an angle 0 with speed v0 then (g = 10 ms–2) : [JEE-Main- 2019]

   5 1 1  2  3
(1) 0  sin1   and v 0  ms (2) 0  cos   and v 0  ms
1

 5 3  5 5

 1  5 1  2  3
(3) 0  cos 1   and v 0  ms
1
(4) 0  sin   and v 0  ms
1

 5  3  5  5
68. A particle of mass m is moving along a trajectory given by [JEE-Main- 2019]
x  x 0  a cos 1t
y  y 0  b sin 2t
The torque, acting on the particle about the origin, at t = 0 is :
 
(1) 
m x 0b22  y 0a 12 k (2) m  x 0b  y 0a  12 k

(3)  my 0a12 k (4) Zero
69. A plane is inclined at an angle   30  with respect to the horizontal. A particle is projected with a
speed u = 2 ms–1, from the base of the plane, making an angle   15 with respect to the plane as
shown in the figure. The distance from the base, at which the particle hits the plane is close to :
(Take g = 10 ms–2) [JEE-Main- 2019]

°
15
=

 = 30°
(1) 18 cm (2) 20 cm (3) 14 cm (4) 26 cm

Corporate Office : B-58, Goal Building, Budha Colony, Patna-1, Help line : 9334594165/66/67 Website : www.goalinstitute.org
Kinematics [ 111 ]
70. A bullet of mass 20 g has an initial speed of 1 ms–1, just before it starts penetrating a mud wall of
thickness 20 cm. If the wall offers a mean resistance of 2.5 × 10–2 N, the speed of the bullet after
emerging from the other side of the wall is close to : [JEE-Main- 2019]
(1) 0.4 ms –1 (2) 0.7 ms –1 (3) 0.3 ms –1 (4) 0.1 ms –1
71. The position vector of a particle changes with time according to the relation
  
 
r  t   15t 2 i  4  20t2 j . What is the magnitude of the acceleration at t = 1?
[JEE-Main- 2019]
(1) 50 (2) 100 (3) 40 (4) 25
72. The position of a particle as a function of time t, is given by x(t) = at + bt2 – ct3 where a, b and c are
constants. When the particle attains zero acceleration, then its velocity will be :
[JEE-Main- 2019]
b2 b2 b2 b2
(1) a (2) a  (3) a  (4) a 
4c 3c 2c c
73. A particle starts from origin O from rest and moves with a uniform acceleration along the positive
x-axis. Identify all figures that correctly represent the motion qualitatively. (a = acceleration,
v = velocity, x = displacement, t = time ) [JEE-Main- 2019]

a v x x
(A) (B) (D) (E)
O t O t O t O t
(1) (A) (2) (A), (B), (C) (3) (A), (B), (D) (4) (B), (C)
   

74. Ship A is sailing towards north-east with velocity v  30 i  50 j km / hr where i points east and j ,
north. Ship B is at a distance of 80 km east and 150 km north of Ship A and is sailing towards west
at 10 km/hr. A will be at minimum distance from B in: [JEE-Main- 2019]
(1) 2.2 hrs. (2) 4.2 hrs. (3) 3.2 hrs. (4) 2.6 hrs.

Answer Exercise - 6
1. (1) 2. (4) 3. (3) 4. (2) 5. (1) 6. (2)
7. (1) 8. (4) 9. (2) 10. (1) 11. (1) 12. (2)
13. (3) 14. (1) 15. (2) 16. (2) 17. (4) 18. (1)
19. (4) 20. (3) 21. (4) 22. (4) 23. (3) 24. (4)
25. (2) 26. (2) 27. (3) 28. (1) 29. (1) 30. (1)
31. (4) 32. (1) 33. (2) 34. (3) 35. (3) 36. (1)
37. (4) 38. (4) 39. (2) 40. (2) 41. (3) 42. (3)
43. (2) 44. (2) 45. (2) 46. (3) 47. (1) 48. (4)
49. (1) 50. (4) 51. (3) 52. (4) 53. (3) 54. (1)
55. (3) 56. (4) 57. (3) 58. (1) 59. (3) 60. (1)
61. (4) 62. (1) 63. (4) 64. (4) 65. (4) 66. (2)
67. (3) 68. (3) 69. (2) 70. (2) 71. (1) 72. (2)
73. (3) 74. (4)

Corporate Office : B-58, Goal Building, Budha Colony, Patna-1, Help line : 9334594165/66/67 Website : www.goalinstitute.org

You might also like